NAVLE Daily Questions

Ace your homework & exams now with Quizwiz!

E - Left caudal lung lobe abscess This dog has a left caudal lung lobe abscess. Radiographic findings: Thorax- There is a soft tissue opacity mass present in the left caudal lung lobe. The mass is deforming the border between the caudal subsegment of the left cranial lung lobe and left caudal lung lobe on the right lateral projection. There is increased opacity in the pleural space with visible fissure lines indicating a small pleural effusion. The cardiovascular structures are normal in size and shape. The musculoskeletal structures and portion of the abdomen included are normal. These findings could have been compatible with either an abscess or a granuloma. In this case a lobectomy was performed and an abscess was confirmed. Hiatal hernias typically show a mediastinal density or mass that may appear and disappear on repeated radiographs. Secondary aspiration pneumonia can be a problem in a patient with a hiatal hernia.

A 1-year-old spayed female mixed terrier is presented with a one-week history of coughing, lethargy, fever, and inappetence. Based on the history and images, which one of the following choices is the most likely diagnosis? [image] [image] A - Diaphragmatic hernia of ventral liver lobe B - Hiatal hernia C - Spirocerca lupi D - Heartworm disease E - Left caudal lung lobe abscess

D. CN VII This is the facial nerve which is responsible for motor of the facial muscles as well as sensation in the ear and lacrimation. CN V is mostly responsible for sensation. CN III is the oculomotor nerve and serves in movement of the eye. CN VI is also involved in movement of the eye.

A 10 year male castrated German Shepherd dog presents for an acute onset of ptosis, drooping of the lip, and drooping of the cheek all on the left side. What nerve has been damaged? A. CN VI B. CN V C. CN III D. CN VII

C - Acer rubrum (Red maple) Ingestion of Acer rubrum (Red maple) is most likely the cause of hemolytic anemia and the resultant pathology in this horse. Wilted or dry leaves and bark of red maple trees contain oxidants which damage red blood cell (RBC) membranes, resulting in the formation of Heinz bodies and eccentrocytes. Acute intravascular hemolysis, hemoglobinemia, methemoglobinemia, hemoglobinuria, and methemoglobnuria follow. Clinical signs of toxicity include: weakness, depression, tachypnea, tachycardia, icterus, cyanosis, and brown discoloration of blood and urine. Post mortem findings include: icterus, splenic hemosiderosis, splenomegaly, swollen dark red-blue black kidneys with red-brown tubular casts, and a swollen brown liver. Toxicity is often fatal. If the horse survives the hemolytic crisis, renal failure may develop due to pigmenturia. Treatment is supportive care: IV fluids, oxygen, and blood transfusions.

A 10 year old Quarterhorse gelding pastured in the northeastern United States is presented for weakness and depression. The owner reports that the horse's urine is an unusually dark brown color. On physical exam, the horse is tachycardic and tachypneic. The mucous membranes are a dark yellowish brown. The horse's plasma has a pink tinge. A CBC, blood smear, biochemistry panel, and urinalysis indentify intravascular hemolytic anemia, hemoglobinemia, methemoglobinemia, and hemoglobinuria. Erythrocytes are seen on the blood smear with Heinz bodies and eccentrocytosis. Ingestion of which one of the following plants is most likely to cause of this spectrum of clinical signs in this case? A - Tetradymia spp (Horsebrush) B - Quercus spp (Oak) C - Acer rubrum (Red maple) D - Veratrum spp (Skunk cabbage) E - Xanthium spp (Cocklebur)

D. Blood pressure The correct answer is blood pressure. Cats with hyperthyroidism are likely to develop hypertension. If this is severe enough (>180-200 mmHg systolic), they can be at risk for acute retinal detachment or hemorrhage resulting in blindness. Prompt resolution of the hypertension is critical to prevent further damage to the eye and other organs.

A 10-year old male castrated cat that you have previously diagnosed with hyperthyroidism presents to you for acute onset of blindness. You perform an ophthalmic exam and note retinal hemorrhage. What diagnostic test should you perform first? A. Coagulation times B. Free T4 levels by equilibrium dialysis C. Serum BUN and creatinine D. Blood pressure E. Total T4 levels

A - Normal finding It is normal for a horse to expel the placenta "inside out", with the allantoic side of the allantochorion exposed. Following parturition, the placenta should be expelled within 3 hours, or it is considered retained in the horse.

A 5-year old Arabian mare expels the placenta 2 hours after foaling. On examination, it appears that the placenta has passed "inside out", with the allantoic side of the allantochorion exposed. What is the correct interpretation? A - Normal finding B - Sign of placental insufficiency C - Suggests premature placental separation D - Do not rebreed this mare E - Foal at risk for peripartum asphyxia

F. Fecal float Hookworms (Ancylostoma and Uncinaria) are intestinal parasites that suck blood and can cause anemia, enteritis, coughing during larval migration, and dermatitis. Any young dog that is failing to thrive and/or has pale mucous membranes should be tested for intestinal parasites. Hookworm dermatitis, also called Ancylostomiasis is typically seen in conditions with poor sanitation and/or in kennels. Hookworms can be transmitted in utero, during nursing, or via 3rd stage larva penetrating the skin. The most commonly affected skin areas are the pads and interdigital spaces of the feet, but can include any surface that contacts the ground. The larva migrate through the dog's tissues before arriving in the intestines. They cause significant anemia, failure to thrive, or sudden death in young dogs. In mild cases, deworming protocols are often enough; with severe cases blood transfusions and parenteral treatments are often necessary. Skin scraping and impression smears of the affected skin areas are typically unrewarding for isolating parasites. A PCV/TP would show signs of anemia, but not the underlying etiology. Complete blood cell count and chemistry will often show anemia that is regenerative and an eosinophilia. Radiographs would be unrewarding in this case. A biopsy may show migrating larva if biopsied soon after trauma, however this is not a diagnostic test routinely used.

A 6-month old Chocolate Labrador presents for limping and failure to gain weight. He is housed in an outdoor kennel with other hunting dogs. He is fed a large breed dry puppy food. He received his puppy shots at 8, 12, and 16 weeks old. He is quiet, alert, and responsive. His body condition score is 3/9, with rib exposure and poor fat deposition. His mucous membranes are pale pink, with a capillary refill time of 2 seconds. His heart and lungs auscult normally. No abnormalities are felt on abdominal palpation. He is an intact male, and both testicles are descended. The only abnormalities are the pads of his two front feet and left hind foot (see image) that he chews at frequently. What is diagnostic test of choice? A. Skin scraping B. Biopsy C. CBC and chemistry panel D. Radiographs E. PCV and TP F. Fecal float

B. Portal systemic shunt Ptyalism is a sign commonly seen with PSS in cats but not dogs, and the copper-colored iris is a striking and almost pathognomonic finding in conjunction with other clinical findings. Hemolytic anemia could cause icterus but not the change in iris color. Animals with polycystic kidney disease would not have a copper-colored iris. Toxoplasma can cause ocular signs such as uveitis but would not have a copper iris.

A 6-month old cat presents for having ptyalism and for being underweight. On physical examination, you note a bright copper color to the cat's iris bilaterally. What is this suggestive of? A. Toxoplasmosis B. Portal systemic shunt C. Hemolytic anemia D. Polycystic kidney disease

A. Treatment is unlikely to be successful and the cow should be culled This is a case of Actinomyces bovis or "lumpy jaw". The keys to the diagnosis are the firm mass that is immobile over the mandible. The presence of "sulfur granules" in the discharge is also a characteristic finding with this disease. Unfortunately, simply making the diagnosis is not sufficient to answer this question correctly. All of the answer choices are potential treatments for lumpy jaw but this represents a moderate to severe case which is unlikely to respond to treatment due to difficulty in achieving the necessary antibiotic concentration over a sustained period. Therefore, the best answer choice in this case is to cull the cow.

A 6-year old Friesian cow presents to you with a mass on the left mandible (see image). The farmer reports that the mass has developed over the last several weeks and the cow has recently had some difficulty eating and lost weight. On examination, the mass is firm, immobile, and painful on manipulation. You note a thick discharge with small granular particles. Based on the most likely diagnosis, what should you tell the farmer? Image used with permission, from Veterinary Cytology (Freeman), courtesy of Manson Publishing. A. Treatment is unlikely to be successful and the cow should be culled B. Intravenous penicillins are likely to be effective C. Treatment with sodium iodide intravenously is likely curative but the cow's milk and meat will not be suitable for human consumption for 120 days D. The most effective treatment is oral sodium iodide

C. Hypothyroidism The clinical signs, physical exam findings, and lab work abnormalities are classical for hypothyroidism. Other common abnormalities seen with hypothyroidism include pyoderma, neuromuscular signs (ataxia, knuckling, vestibular signs, etc), markedly elevated triglycerides, and a mild normocytic, normochromic anemia. Hyperthyroidism rarely occurs in dogs. You would expect to see polyuria, polydipsia, and polyphagia with hyperadrenocorticism.

A 6-year old female spayed English Pointer presents for lethargy and weight gain. The owner notes that the dog is eating and drinking a normal amount, but the dog is still gaining weight. A physical exam reveals weak pelvic limbs, facial nerve paralysis, a symmetrically patchy haircoat, and seborrhea. Lab work reveals a normocytic, normochromic anemia with a PCV of 29% (35-57%), lipemic serum, and cholesterol of 1090 mg/dl (135-278 mg/dl). What is the most likely diagnosis? A. Hyperthyroidism B. Adrenal dependent hyperadrenocorticism C. Hypothyroidism D. Pituitary dependent hyperadrenocorticism

A. Immunosuppressive therapy and a change to a hypoallergenic diet

A 6-year old female spayed indoor only domestic medium hair presents with a large raw lesion on its upper left lip. The owner reports the lesion has come and gone over the past few years. Which of these treatments would likely be most helpful? A. Immunosuppressive therapy and a change to a hypoallergenic diet B. Topical trifluridine and systemic clindamycin C. Marginal surgical excision and radiation therapy D. Wide surgical excision and chemotherapy

B. Osteosarcoma Chondrosarcoma, fibrosarcoma and hemangiosarcoma can all be primary bone tumors but are much less common in dogs than osteosarcoma.

A 6-year old male neutered Weimaraner presents for left forelimb lameness. Radiographs are shown below and show a mixed productive and destructive lesion affecting the left distal radius with accompanying soft tissue swelling. The lesion does not cross the joint. What is the most common primary bone tumor in the dog? A. Hemangiosarcoma B. Osteosarcoma C. Fibrosarcoma D. Multiple myeloma E. Chondrosarcoma

C. Prednisone This is a case of lymphoma. The cytology depicts the classic finding of a population of lymphoid cells that are predominantly lymphoblasts based on their size and characteristics. If you were unsure about their size due to magnification, there is a neutrophil in the lower left corner for comparison. The lymphoblasts are considerably larger than the neutrophil. Mature lymphocytes would be smaller than a neutrophil. Note that there is a mitotic figure in the middle of the slide. There are many treatments and protocols for lymphoma and some of the main agents known to have efficacy are prednisone, doxorubicin, cyclophosphamide, vincristine, L-asparaginase, and lomustine. There are many other efficacious chemotherapeutics for lymphoma but carboplatin is not considered a first line treatment for lymphoma in dogs. The other drugs listed are antifungal (itraconazole), antibiotic (doxycycline) and anti-parasitic (milbemycin).

A 7-year old female Lhasa Apso presents to you for lethargy and inappetence. On your exam, you detect mandibular lymphadenopathy and perform a fine needle aspirate. You see the aspirate depicted here. Which of these treatments would be given to this patient as part of a first line therapy? A. Milbemycin B. Doxycycline C. Prednisone D. Itraconazole E. Carboplatin

B - ACTH stimulation. Sudden-onset blindness in an older dog with normal fundi suggests Sudden Acquired Retinal Degeneration (SARD). Typically, SARD is associated with metabolic abnormalities-PU/PD, polyphagia, obesity and hyperadrenocorticism. An ACTH stimulation test (or an LDDS) can help you determine if the dog has hyperadrenocorticism and SARD. SARD and progressive retinal atrophy (PRA) are often categorized together as forms of retinal degeneration.

A 9 year old female spayed Brittany is presented with a 1 week history of apparent disorientation and bumping into things around the house. The owners report recent weight gain, increased appetite, increased water-drinking and frequent urinary accidents. The dog is obese but otherwise appears healthy and afebrile. The pupils are dilated and poorly responsive to light. The ocular fundus appears normal OU (both eyes). In addition to routine labwork (CBC, biochemistry panel, urinalysis) what other diagnostic test should be considered? A - Genetic testing for rod-cone photoreceptor dysplasia B - ACTH stimulation C - Serum, urine ornithine concentration D - Gradual water deprivation test E - Tonometry

C - Norway rat The rat has the best prognosis. MAMMARY GLAND TUMORS are typically BENIGN in rats (but MALIGNANT in mice). As many as 50% of boid snakes (boas, pythons) harbor the retrovirus which can cause Inclusion Body Disease (IBD). IBD can cause regurgitation, weight loss and in later stages neurologic signs, like failure to right itself. Febrile catarrhal (mucopurulent oculonasal discharge) in a ferret is suggestive of Canine Distemper (poor Px).

A clinic's waiting room has: A Rottweiler dog with foreleg osteosarcoma A white mouse with a mammary gland tumor on her belly A Norway rat with a mammary gland tumor near her neck A snot-nosed ferret with a chin rash and crusts around the eyes A thin boa constrictor that can't right itself from dorsal recumbency Which one of these animals has the BEST prognosis? A - Ferret B - White mouse C - Norway rat D - Rottweiler E - Boa constrictor

A - Rumenal infusion 2-8 liters vinegar,3-10 gallons cold water Treat Urea/non-protein nitrogen (NPN) toxicity with a rumenal infusion 2-8 liters 5% acetic acid (vinegar) and 3-10 gallons of cold water. The vinegar decreases rumenal pH which slows absorption of un-ionized ammonia. Repeat Q 6 hours up to 48 hours. Best results if animal is still ambulatory. It is often impossible to treat these cases before they die because of rapid progression to death. If possible, Rx with IV fluids. If necessary, relieve bloat. Urea/NPN toxicity is related to ammoniated feed toxicity which causes so-called "Bovine bonkers". Manage ammoniated feed toxicity by removing the ammoniated feed and treating severe cases with anticonvulsants like diazepam, pentobarbital.

A cow is presented on emergency with urea/non-protein nitrogen toxicity. What is the treatment of choice? A - Rumenal infusion 2-8 liters vinegar,3-10 gallons cold water B - Relieve bloat, drench with 2-8 liters sodium bicarbonate C - Rumenotomy D - IV Fluids with MgSO4, Na thiosulfate PO E - Atropine, Protopam chloride IV q 4-6 hours

B - 34.5 Anion gap= (Positives)-(Negatives) = (Na+ and K+) -(Cl- and HCO3-). If you did not do a blood gas analysis, then substitute TCO2 for HCO3-, and the AG value is about the same, (34.2 in this case). Anion Gap (AG) = [136.2 (Na+) + (3.5(K+)] - [91.3 (Cl-) + 13.9(HCO3-)] AG= [139.7 cations] - [105.2 anions]=34.5 anion gap Typically see increased anion gap with: Diabetic ketoacidosis Renal insufficiency Ethylene glycol toxicity Lactic acidosis from grain overload or strenuous exercise Decreased anion gap is UNcommon. Think hemodilution, hypoalbuminemia, hypercalcemia.

A dog is presented with the following findings. Blood Gas PCO2 venous=28.3 mm Hg....[N=35-44], pH=7.27..[N=7.31-7.53] PO2 arterial=85.2 mm Hg....[N=85-95] HCO3-=13.9 mEq/l..[N=25-35] Blood chemistry Na=136.2 mEq/L.................[N=146-156], Cl=91.3 mEq/L..[N=109-122] Ca=9.1 mEq/L.....................[N=9.6-11.6], K=3.5 mEq/L..[N=3.8-5.6] P=9.3 mEq/L.......................[N=2.5-6.2], TCO2=14.2 mEq/L..[N=17-27] ALT=331 U/L...................[N=8.2-57], LDH=354 U/L..[N=24-219] AST=14 U/L....................[N=13-15], SDH=6 U/L..[N=3.1-7.6] GGT=20 U/L....................[N=1.0-9.7], Bilirubin (total)=0.3 mg/dl....[N=0.0-0.4] Alk Phos=1018 U/L..........[N=1-114 U/L] BUN=28 mg/dL (10 mmol/L)..........[Normal: 7.0-26 mg/dl (2.5- 9.29 mmol/L)] Glucose=150 mg/dl or 8.3 mmol/L.........[N 63-132 g/dl or 3.5-7.3 mmol/L] What is the anion gap? A - 28.1 B - 34.5 C - 28.3 D - 31.2 E - 42.2

B. Pseudomonas The correct answer is Pseudomonas. Pseudomonas is the most common agent causing bacterial keratitis in the horse. The gram negative cytology provides further evidence that it is the likely culprit. Staphylococcus can cause keratitis in horses but is gram positive. E. coli and Pasteurella are not agents that are commonly involved with keratitis in the horse.

A horse presents to you with a corneal ulcer. You are concerned because it appears to be infected, as shown in this image. You perform cytology and find gram negative rods. What is the most likely organism infecting the corneal ulcer in this horse? Image Courtesy of Brad Holmberg, DVM MS PhD DACVO A. E. coli B. Pseudomonas C. Pasteurella multocida D. Staphylococcus spp.

B - Improve sanitation, install smooth-floored enclosure Bumblefoot (pododermatitis) in guinea pigs usually occurs secondary to poor sanitation, obesity and wire cage floors or rough bedding. If detected early, switch to smooth-bottom flooring, keep the enclosure clean, and change to a softer bedding. Chlorhexidine soaks and debridement can help the feet. Prognosis is guarded. Avoid penicillins in guinea pigs.

A large male guinea pig from a group of four is presented with swollen and scabby hind feet from which Staphylococcus aureus is cultured. Which one of the following recommendations is the most appropriate choice? A - Isolate affected animal, treat with oral amoxicillin B - Improve sanitation, install smooth-floored enclosure C - Cull affected animal, increase ventilation of environment for remaining animals D - Tetracycline-medicated water for all animals E - Debridement and topical 1% butenafine cream

A - Dilated cardiomyopathy This is a classic presentation for dilated cardiomyopathy. Prognosis is GUARDED- with Rx may live 6 mos-2 years. Px especially poor for Dobies. Rx varies, but foundation includes the "3 D's" -- Diet (Low salt), vasoDilators (Enalapril), Diuretics (Furosemide), +/- Digoxin (+ inotrope to strengthen contraction). NOTE that Dobies can be sensitive to digitalis, so may need lower doses. So, remember your "DCM D's" : "Depressed Dilated Dobies need Diet, Dilators, Diuretics (and maybe Digoxin, but overDose is Doom)."

A middle-aged, depressed, coughing, exercise-intolerant Doberman pinscher is presented. The dog has a rapid and irregular heart rate. What disease is at the top of the differential diagnosis list? A - Dilated cardiomyopathy B - Mitral regurgitation C - Left-sided congestive heart failure D - Wolf Parkinson White syndrome E - Tricuspid insufficiency

D - Normal in a neonatal foal These findings are NORMAL in a neonatal foal. The heart rate and a finding of a continuous murmur on the left side (due to slight opening of the ductus arteriosus, which usually closes within 4-5 days) are normal in newborn foals. Persistent patent ductus arteriosus is rare in horses.

A newborn foal is examined two hours after birth. The foal has a heart rate of 100 bpm [N=100-120], and a continuous murmur loudest on the left side. Which one of the following choices is the most likely explanation for these findings? A - Incipient septicemia B - Hypoxic ischemic encephalopathy C - Foal dysmaturity D - Normal in a neonatal foal E - Ventricular septal defect

C. Tyzzer's disease This is the most likely cause because of the age of the foal and the acute nature of the disease. Tyzzer's disease is caused by Clostridium piliformis, which causes an acute necrotizing hepatitis. It affects only foals from about 1-6 weeks of age. Theiler's disease is a condition of adult horses. Clostridium novyi is rare in horses and seen much more in sheep and cattle. Herpesvirus can cause hepatitis but is usually seen at or very soon after birth.

A one-month old foal develops fever, icterus, and diarrhea acutely. Bloodwork shows hyperfibrinogenemia, hypoglycemia, and elevated liver enzymes. Which of these conditions is most likely? A. Clostridium novyi type B B. Theiler's disease C. Tyzzer's disease D. Herpesviral hepatitis

C. Furosemide and enalapril The radiograph here shows a severe symmetrical alveolar pattern in the perihilar region extending to the right and left caudal lung lobes. The heart is tall on the lateral view causing dorsal elevation of the trachea. This, in conjunction with the physical findings, is compatible with left congestive heart failure (CHF) secondary to mitral valve regurgitation. The dog also has hepatic congestion evident by enlargement on the radiograph. Treatment for CHF include diuretics (furosemide/Lasix, hydrochlorothiazide, spironolactone), arterial vasodilators (enalapril, benazepril, amlodipine, hydralazine), positive inotropes (pimobendan), and venodilators (nitroglycerine). Clavamox and enrofloxacin are antibiotics which could be used to treat pneumonia, but that is not this dog's problem. Immiticide is the treatment for heartworm. Atenolol is a beta blocker and not part of the management of mitral regurgitation or CHF. Atropine or a pacemaker would be indicated for conduction problems.

An 11-year old female Pomeranian presents to you for coughing and exercise intolerance. On exam: Wt: 9.25 lbs, T: 101.2F (38.4 C), HR: 132 bpm, RR: Panting, mucous membranes are pink. She has mild tracheal sensitivity and a grade III-IV/VI left apical holosystolic murmur and grade II/VI right apical holosystolic murmur. Femoral pulses are strong and synchronous, with a regular rhythm. She has harsh lung sounds bilaterally. On abdominal palpation, you note hepatomegaly. You find bilateral luxating patellas. You take chest radiographs (see image) and decide to treat the dog based on these findings. Which medication plan is most appropriate? A. Immiticide (Melarsomine) B. Oxygen and atenolol C. Furosemide and enalapril D. Atropine and a temporary pacemaker E. Clavamox and enrofloxacin

D - Pasteurella multocida Pasteurella multocida infection in conjunction with Mycoplasma hyopneumoniae causes exudative bronchopneumonia, polyarthritis, and chronic lung lesions. Mycoplasma hyopneumoniae, (also called "Enzootic pneumonia") is a common, smoldering low-level illness. Stressors (parasites, other infections, even weather) can result in severe pneumonia. Best managed by decreasing stressors with improved ventilation and decreased overcrowding. In endemic herds, ABX for sick individuals (ie: lincomycin, tylosin, tiamulin, or a tetracycline) helps control illness, most likely by preventing secondary bacterial infection (like Pasteurella). Bacterin vaccines give good protection, decrease signs (coughing). Pre-farrowing vaccination of sows decreases colonization in suckling piglets.

Failure to control mild endemic respiratory disease in swine caused by Mycoplasma hyopneumoniae predisposes pigs to complications. Exudative bronchopneumonia and polyarthritis are most commonly seen in herds infected with both Mycoplasma hyopneumoniae and which other organism? A - Bordetella bronchiseptica B - Fusobacterium necrophorum C - Hemophilus parasuis D - Pasteurella multocida E - Swine influenza virus

D - Parvovirus. Expect to see intestinal crypt necrosis, lymphoid depletion of Peyer's patches, shortened, blunt villi and collapse of the lamina propria with canine parvovirus. Puppies with parvo are severely neutropenic and lymphopenic which may help you remember that feline panleukopenia is ALSO caused by a parvovirus. Think of canine distemper with intracytoplasmic eosinophilic inclusion bodies, catarrhal enteritis and mucopurulent oculonasal discharge. Canine adenovirus-1 is the causative organism of infectious canine hepatitis. NOTE: If you hear of necrotic Peyer's patches from necropsy of a cow with severe diarrhea, stomatitis and fever think of the foreign disease, rinderpest. In 2011, the United Nations Food and Agriculture Organization (FAO) and the World Organisation for Animal Health (OIE) officially declared that rinderpest was eradicated globally. But because it is a classic, severe, reportable, stomatitis-type disease, it's unlikely that vets will be allowed to forget rinderpest on DDXs for years. Click here to see images of bovine rinderpest.

Histopathology findings show intestinal crypt necrosis and lymphoid depletion of Peyer's patches. The sample came from a 4 month old male Rottweiler puppy that died following a severe illness with vomiting, fever and neutropenia. Which one of the following choices is the most likely diagnosis? A - Enterotoxigenic E. coli B - Thymic hypoplasia C - Distemper D - Parvovirus E - Canine adenovirus-1

B. Left lung On a right lateral radiograph, the right side is down. In this situation, the right lung lobes are compressed and the left lung lobes inflated, accentuating a mass in the left lung. On a left lateral radiograph, the left lobes are compressed and a mass in the left lung can be concealed due to compression of the lung surrounding the mass.

If a mass appears in the lungs on a right lateral radiograph but not on a left lateral radiograph, where is the mass located? A. Body wall B. Left lung C. Mediastinum D. Right lung

E - Miosis, Protruding nictitans, Enopthalmos, Ptosis Remember "My 3rd Sunken Toe" (Miosis, 3rd lid protrudes, Sunken eye, Ptosis) and "sweaty horses". A syndrome, not a disease per se. Can see 4 things with Horner's, ALL associated with the eye: 1. MIOSIS (constricted pupil-lose sympathetic innervation) 2. PROTRUSION 3rd eyelid (nictitans) 3. ENOPHTHALMOS (sunken eye) 4. PTOSIS (drooped eyelid), +/- anisocoria Follow this link to see a dog with Horner's Syndrome OS (left side) Anhidrosis (dry, decreased sweating ipsilaterally) occurs in all animals EXCEPT horse, but difficult to observe. PARADOXICALLY, Horses can show profuse sweating ipsilaterally due to loss of peripheral vasoconstriction.

It is your first day on the job and your boss says there is a dog in room two with Horner's syndrome. Which four signs correctly constitute Horner's syndrome? A - Sweating ipsilaterally, Exopthalmia, Sunken nictitans, Mydriasis B - Dry eye ipsilateral, Miosis, Sunken nictitans, Exopthalmos C - Proptosed nictitans, Lip deviated ipsilaterally, Miosis, Anhidrosis D - Protruding nictitans, Bupthalmia, Ptosis, Mydriasis E - Miosis, Protruding nictitans, Enopthalmos, Ptosis

A. 7-10 days The correct answer is 7-10 days. Canine parvovirus mainly affects puppies and young dogs less than a year of age. Transmission of the virus is through contact with infected feces and fomites such as hands, toys, the dog's hair coat. The virus is very resilient in the environment and is resistant to many types of disinfectants. It replicates in the crypt epithelium of the gut and causes epithelial necrosis and hemorrhagic diarrhea. The virus can also affect the heart of young puppies, causing myocarditis. This occurs less commonly now since most bitches are immunized against the virus, which allows for maternal antibodies to protect young puppies from this form of the disease.

Once infected, for what period of time is canine parvovirus usually shed? A. 7-10 days B. 5-6 months C. 2-3 months D. 3-4 weeks

B. Clostridium perfringens type C The age of these affected pigs along with the rapid course of hemorrhagic and necrotic enteritis help you come to this conclusion. As with many Clostridial diseases, vaccination is the most effective means of control. Other important clostridial diseases of swine include C. perfringens type A, C. difficile, C. tetani, C. botulinum, C. novyi, C. septicum, and C. chauvoei.

Several litters of 2 to 3 day old pigs have recently died rapidly with hemorrhagic enteritis. Post mortem lesions include mucosal hemorrhage, necrosis and emphysema in the small intestines. What pathogen is most likely to cause these signs? A. Haemophilus parasuis B. Clostridium perfringens type C C. Salmonella choleraesuis D. Erysipelothrix rhusiopathiae E. Enterotoxigenic E. coli

E. Low potassium The cat in the image is exhibiting cervical ventroflexion, which is a general sign of weakness. The most common cause for this presentation is hypokalemia (low potassium), which can be caused by a variety of reasons (such as chronic renal failure). This weakness can also be caused by a number of different problems other than low potassium, including myasthenia gravis; polymyopathies caused by toxoplasmosis, immune-mediated disease, or hyperadrenocorticism; and neuropathies caused by organophosphate poisoning, thiamine deficiency, or botulism.

The cat in the image below presents for lethargy, depression, and weakness. The cat can't seem to lift his head (as seen in this image). You recommend running a chemistry panel on the cat. What potential finding explains can explain the findings? Image used with permission, from Feline Medicine (Sparkes), courtesy of Manson Publishing. A. Low calcium B. High glucose C. High urea nitrogen D. Low phosphorus E. Low potassium

A -High sulfur diet This is the cerebrocortical necrosis of polioencephalomalacia (PEM), basically a nutritional disease. PEM is traditionally associated with LOW THIAMINE but increasingly associated with HIGH SULFUR diets. PATHOGNOMONIC dorsomedial strabismus ("Stargazing"), history and cerebrocortical necrosis that lights up under UV light all suggest PEM.

This cow was found recumbent and blind, with dorsomedial strabismus, in tonic-clonic seizures that led to coma and death. On necropsy, lesions shown below light up under ultraviolet light. Click here to see image What caused this problem? A -High sulfur diet B - Rabies C - Bovine spongiform encephalopathy D - Urea toxicity E - Thromboembolic meningoencephalitis (TEME)

C - Salmonella typhimurium Salmonella typhimurium is associated with rectal strictures in growing pigs. Caused by an ulcerative proctitis that damages rectal tissue. Can see large numbers of cases. Can see sporadic rectal strictures as a sequelae to rectal prolapse. Follow this link to see a Merck image of rectal prolapse in a sheep. Intestinal spirochetosis is a post-weaning diarrhea seen in the ABSENCE of Brachyspira (Serpulina) hyodysenteriae (swine dysentery), but similar in presentation to it. This syndrome is being recognized more frequently worldwide.

What condition is associated with rectal stricture in pigs? A - Rotavirus B - Intestinal spirochetosis C - Salmonella typhimurium D - Intussusception E - Coccidiosis

A - Good place to take a pulse A convenient way to take the pulse of a horse is to place two or three fingers over the facial artery, in the notch of the mandible just in front of the masseter (cheek) muscle. Click here to see a to see a video of a vet taking a horse's pulse. There are 4 other places on a horse you can take a horse's pulse, but only one is also on the head: -Transverse facial artery (caudal to lateral canthus of the eye) -Median artery (proximal, medial forearm) -Great metatarsal artery (hindlimb, between McIII and McIV) -Digital artery (caudal pastern area, above hoof). Palpate the digital artery to feel the "bounding pulses" of equine laminitis.

What is the clinical importance of the anatomic area indicated by the red arrow in this image? A - Good place to take a pulse B - Best surgical approach to mandibular sinus C - Upper limit of Viborg's triangle D - Most common site of tooth root abscess E - Site of glossopharyngeal nerve block

D. Squamous cell carcinoma

What is the most common neoplasia seen in the equine stomach? A. Mesothelioma B. Lymphosarcoma C. Gastric adenocarcinoma D. Squamous cell carcinoma

A - Spirocerca lupi A disease of dogs in the Southern U.S. and tropical climates, Spirocerca lupi (esophageal worms) make reactive granulomas of variable size in the esophageal, gastric or aortic walls. Large granulomas may become neoplastic (osteosarcoma, fibrosarcoma). Some dogs develop spondylitis or enlargement of the extremities characteristic of hypertrophic osteopathy. Typically asymptomatic, but large granulomas can cause esophageal obstruction. Spirocercosis may also lead to aneurysm in the thoracic aorta or an ossifying spondylitis of the posterior thoracic vertebrae. Habronema spp. (Habronema muscae, H. microstoma and Draschia megastoma) in horses can cause tumorlike stomach nodules and sometimes cutaneous lesions. Blastomycosis, most common in dogs, cats and humans is characterized by pyogranulomatous lesions in various tissues. Placental infection in horses with Coccidioides immitis have been described, leading to abortion and osteomyelitis.

Which choice is associated with osteosarcoma and hypertrophic osteopathy? A - Spirocerca lupi B - Coccidioidomycosis C - Habronema spp. D - Blastomycosis E - Ollulanus tricuspis

C - Dexamethasone Dexamethasone is allowed in food animals, but may cause abortion in pregnant animals. Typically, use dexamethasone in cattle to induce parturition (20-30 mg, IM, given within 2 wk of normal term). Click here to see a list of drugs prohibited for extra-label use in food animals. Here is an FDA summary on the Ins and Outs of Extra-Label Drug Use in Animals. Diethylstilbesterol (DES) is banned for use in food producing animals and should never be used. Chloramphenicol has been associated with bone marrow suppression/aplastic anemia in exposed humans, and is contraindicated in food-animals. According to a 2006 report from the Food and Drug Administration (FDA) the use of Estradiol cypionate (ECP) in animals is illegal. ECP has been used as an estrogenic hormone for reproductive therapy in food animals, but even extra-label, this is not allowed. Furazolidone a nitrofuran, is not allowed in food animals.

Which choice is permitted for use in food animals? A - Diethylstilbesterol B - Chloramphenicol C - Dexamethasone D - Estradiol cypionate E - Furazolidone

B - Proximal interphalangeal joint (PI-PII) Clinically important because one of the 3 most important nerve blocks, the foot block (also called pastern or abaxial sesamoid nerve block (ASNB)) is done just above the pastern. Can use a pastern block to make a horse with acute laminitis comfortable. Follow this link to see a horse "leaning back" in classic laminitis posture. Refs: Pasquini & Spurgeon's Anatomy Dom An 11th ed. p. 112-7 587-92 and Pasquinis, Jahn & Bahr, Guide to Eq Clin: LAMENESS vol. II, p. X8 and the Merck Veterinary Manual online edition.

Which choice is the pastern joint? A - Carpometacarpal joint B - Proximal interphalangeal joint (PI-PII) C - Distal interphalangeal joint (PII-PIII) D - Metacarpophalangeal joint E - Intermetacarpal joint

C. Streptococcus suis The correct answer is Streptococcus suis. Streptococcus suis and H. parasuis both cause polyarthritis, polyserositis, fever, and pneumonia in young piglets up to several weeks of age. Both can cause fibrinopurulent inflammation as well as meningitis and convulsions. Erysipelas and M. hyosynoviae typically occur in grower and finisher pigs and do not result in pneumonia. Diamond-shaped skin lesions (thus the name "diamond skin disease") are pathognomonic for Erysipelas. Fusobacterium necrophorum causes lameness via footrot or laminitis.

Which infectious agent causes the clinical presentation of pigs most similar to Haemophilus parasuis (Glasser's disease)? A. Fusobacterium necrophorum B. Mycoplasma hyosynoviae C. Streptococcus suis D. Erysipelothrix rhusiopathiae

B - White on right arm, black on left arm, red on left leg. The electrodes used to record an ECG are always attached the same way. The white colored wire is attached to right arm, black to the left arm, and red to the left leg. When 4 are used, the green wire is attached to the right leg. To record each lead (I, II, III), the ECG monitor automatically compares the electrical difference between the appropriate locations. The different leads allow measurement of activity from different angles around the heart. This is useful for monitoring, as different leads are tried till a good waveform is found. The waveforms vary with each lead, abnormal waveforms are seen with cardiac enlargement.

Which of the following correctly describes placement of the electrodes when performing an electrocardiogram on a dog or cat? (note arms = forelegs, legs = rear legs) A - White on left arm, black on right arm, red on right leg B - White on right arm, black on left arm, red on left leg C - Red on right arm, green on left leg, black on left arm D - Black on right arm, white on left leg, red on right leg E - Red on left leg, green on right arm, black on left arm

B. Recipient red blood cells, donor plasma In a minor crossmatch, you are looking to see if the factors in the plasma of the donor are going to react to the recipient's red blood cells.

Which of these are used in the minor cross match for blood products when looking for a compatible blood donor for a dog? A. Recipient red blood cells, recipient plasma B. Recipient red blood cells, donor plasma C. Donor red blood cells, donor plasma D. Donor red blood cells, recipient plasma

C. Auriculopalpebral nerve block Blocking this branch of cranial nerve VII (facial nerve) disrupts the motor innervation to the orbicularis oculi, which is the muscle that closes the eye. In the horse, this muscle is very strong and can prevent a thorough ocular exam. To perform this block, lidocaine is injected subcutaneously at the caudal aspect of the zygomatic arch where the nerve is palpable. Because this is a motor nerve, the eye will not be able to close as well, but there is no anesthesia of the tissue.

Which of these nerve blocks is used to examine the eyes of a horse? A. Trigeminal nerve block B. Corneal nerve block C. Auriculopalpebral nerve block D. Oculomotor nerve block

A - Tetracycline Long acting oxytetracyclines are effective against Campylobacter jejuni and C. fetus subsp fetus, the organisms that cause vibriosis. Daily dihydrostreptomycin or penicillin can also be used but is more labor intensive. Chloramphenicol is prohibited in food animals.

Which one of the following antibiotics is the treatment of choice in the face of a vibriosis abortion storm? A - Tetracycline B - Aminoglycoside C - Cephalosporin D - Fluoroquinolone E - Chloramphenicol

D - Pull a blood sample for a Western blot FIV test Pull a blood sample for a Western Blot FIV test. The feline immunodeficiency virus (FIV) ELISA is the standard first-step screening test to detect if the cat has been exposed to the FIV virus and has circulating antibody. A confirmatory Western blot antibody test for FIV is the standard confirmatory test. Note - New PCR tests show promise for confirming FIV infection regardless of vaccination status, but are not yet the standard of clinical practice. PCR was not among the answer choices in this question to avoid confusion, and to mimic the real test. On the real exam, you should NOT expect to see equivocal, controversial or cutting-edge new treatments or diagnostic tests. A Western blot test is especially important in areas with low FIV prevalence, where the risk of false positive FIV ELISA is higher. Click here to see table that shows why a test's predictive value goes down as prevalence goes down. Remember that cats vaccinated for FIV will test positive for FIV antibodies by FIV ELISA and Western blot. Kittens up to 6 months of age born to seropositive queens can also be seropositive, even though they are not infected, due to persistence of maternal antibodies. Seropositive kittens should be retested at greater than 6 months of age. This is DIFFERENT from feline leukemia virus (FeLV) testing because the FeLV ELISA and IFA tests measure ANTIGEN, not antibody, so FeLV vaccination does NOT interfere with testing.

Which one of the following choices is the most appropriate next step after a 3-year old stray cat tests positive for Feline Immunodeficiency Virus (FIV) on a routine FIV ELISA screening test? A - Recheck serum by FIV complement fixation test B - Do a Rivalta test to rule out feline coronavirus-related interference C - Pull a blood sample for FeLV Antigen test D - Pull a blood sample for a Western blot FIV test E - Euthanize

E - Ancylostoma spp. Hookworms (Ancylostoma spp.) may cause cutaneous larva migrans in people. Note that Roundworms (Toxocara spp., Toxasacaris spp.) are also zoonotic, causing visceral and ocular larva migrans in people. In dogs, Whipworms, Trichuris spp. are associated with a hypoadrenocorticism-like syndrome (hyponatremia, hyperkalemia, azotemia, metabolic acidosis). Whipworm infection has been suggested as one cause of cecocolic intussusception. Habronema spp. in horses can cause tumorlike stomach nodules and sometimes cutaneous lesions. Trichostrongylus spp. cause parasitic gastritis and enteritis in sheep, goats, and cattle.

Which parasite can cause cutaneous larva migrans in people? A - Habronema spp. B - Trichuris spp. C - Spirocerca sp. D - Trichostrongylus spp. E - Ancylostoma spp.

A - Cauda equina neuritis, herpesvirus myeloencephalopathy Horses with cauda equina neuritis (also called polyneuritis equi) have a progressive symmetric LMN paresis of the tail, bladder, rectum, anal sphincter. Look for urinary incontinence, fecal retention, and a weak or paralyzed tail. May see hind limb paresis if lumbosacral spinal cord is affected. Cranial nerves can also be affected, but typically cranial involvement is asymmetric. May see temporal or masseter atrophy (cranial nerve 5), facial paralysis, and exposure keratitis (cranial nerve 7), head tilt, or other CNS signs. Cause is unknown, may be an autoimmune process. Grave prognosis. Eventually euthanized. Herpesvirus myeloencephalopathy (EHV-1) may also present with urinary incontinence, but this is a relatively uncommon manifestation of equine rhinopneumonitis. You would expect to hear a history of the more common EHV signs in other horses from the same farm, like respiratory disease ("snots") in foals and abortions in mares. Bracken fern toxicity (Pteridium aquilinum) causes thiamine deficiency in monogastrics (like horses) and bone marrow depletion, aplastic anemia, and bladder tumors in ruminants. In horses, look for signs of anorexia, weight loss, incoordination, and a crouching stance with feet placed wide apart. Horses may have trembling muscles when forced to move.

Which two diseases would be on a differential diagnosis list for equine urinary incontinence? A - Cauda equina neuritis, herpesvirus myeloencephalopathy B - Equine degenerative myelopathy, botulism C - Nigropallidal encephalomalacia, locoweed poisoning D - Bracken fern toxicity, equine encephalomyelitis E - Tetanus, West Nile virus encephalopathy

A. Penicillin Erysipelas is susceptible to penicillins, as well as tetracyclines (usually), lincomycin and tylosin. Chloramphenicol and nitroimidazoles (including metronidazole) are not approved for food animal use.

You are asked to examine some feeder pigs that have stopped eating yesterday. The group is lying down and seems lethargic. They have fevers of 105-106F (40.6 -41.1 C), firm dry feces, and the skin has rhomboid-shaped red blotches scattered on it. What treatment should be recommended? A. Penicillin B. Gentamicin C. Streptomycin D. Metronidazole E. Chloramphenicol

B. 4 to 8 liters of fresh whole blood IV In the case of an acute blood loss such as this, the most important treatment is whole blood. Other sodium-containing fluids may be beneficial while the blood is being collected if this will not result in losing time in getting the whole blood into this cow. In most cases the single blood transfusion results in recovery. After about 24 hours the cow will exhibit melena as the digested blood reaches the rectum

You are called to examine and treat a valuable 3 year old show cow that appeared normal yesterday but has collapsed and is too weak to get up (see photo). You examine her and find T=102F or 38.9 C, HR=130, and RR=42. Her heart is pounding very loudly. The mucous membranes of her eyes, mouth and vulva are all very pale. You diagnose an acutely bleeding abomasal ulcer. What is the most important treatment? A. 20 liters of 1.3% sodium bicarbonate IV B. 4 to 8 liters of fresh whole blood IV C. 40 liters of saline IV D. 4 liters of commercial plasma IV plus vitamin K E. 40 liters balanced electrolytes orally

B. Dermanyssus gallinae is controlled primarily by thorough insecticidal treatment of the environment and Ornithonyssus sylviarum must be controlled by application of approved pesticides to affected birds Dermanyssus gallinae (the common red mite) maintains the majority of the population in the environment so it is important to monitor and treat the environment including the cracks and crevices where mites tend to reside. Ornithonyssus sylviarum mites (northern fowl mites) spend their entire life cycle on the bird. This allows them to multiply more rapidly and makes it essential to treat the birds themselves to control the problem.

You are visiting a small "back-yard" flock of chickens and examining the animals. You find several small gray-red mites (approx 0.7 mm) on several of the chickens which you recognize as Dermanyssus gallinae and Ornithonyssus sylviarum. How are these mites best controlled? A. Ornithonyssus sylviarum is controlled primarily by thorough insecticidal treatment of the environment and Dermanyssus gallinae must be controlled by application of approved pesticides to affected birds B. Dermanyssus gallinae is controlled primarily by thorough insecticidal treatment of the environment and Ornithonyssus sylviarum must be controlled by application of approved pesticides to affected birds C. Control of both Ornithonyssus sylviarum and Dermanyssus gallinae relies on application of approved pesticides to affected birds D. Control of both Dermanyssus gallinae and Ornithonyssus sylviarum relies on thorough insecticidal treatment of the environment

C. 60 ml In order to raise the PCV 1% you will need to give 1ml/kg of packed red blood cells. So it takes 6mls to raise this cat's PCV by 1%. If we are going to increase it by 10% we will need 60ml of packed red blood cells.

You have a 6-kg cat that you wish to raise his PCV from 15 to 25%. You plan to administer packed red blood cells. How many milliliters of packed red blood cells will this cat need? A. 15 ml B. 120 ml C. 60 ml D. 30 ml

C. Benign Neglect Histiocytomas are typically benign and will often present as a small, raised mass that may or may not be ulcerated. Fortunately, these masses usually regress on their own, and surgical or medical intervention is typically not necessary.

A 2-year old female spayed Dachshund presents for further evaluation as a result of a mass on the pinna noted by the owner. On physical exam, the only abnormality noted was this mass. Cytology is consistent with a histiocytoma. Which of the following is the most reasonable treatment option? A. Radiation therapy B. Radical surgical excision C. Benign Neglect D. Cryotherapy E. Chemotherapy using lomustine

A - Panosteitis The hallmark of panosteitis is an acute-onset shifting leg lameness with long bone pain in a young (5 mos-2 years), medium to large breed dog. German Shepherds are reported to be at highest risk of panosteitis. Basset hounds may be over-represented. Hypertrophic osteodystrophy (HOD) is characterized by bilateral metaphyseal pain, swelling in distal radius and ulna, fever, anorexia and depression. Typically a disease of young, growing large and giant breed dogs. Hypertrophic osteopathy (HO) is a diffuse periosteal proliferative disease of long bones in dogs SECONDARY to neoplastic or infectious masses in the thoracic or abdominal cavity. Multiple cartilaginous exostoses is characterized by non-painful ossified swellings on metaphyseal cortical surfaces of long bones, vertebrae, and ribs in young dogs and cats. Animals may have no signs at all. Diagnosis is confirmed by palpation and radiography. Click here to see a radiograph of cartilaginous exostoses Retained ulnar cartilage cores are characterized by lameness and eventually, angular limb deformity in young, large breed dogs.

A 1 year old German Shepherd is presented with a 2-week history of lameness. Although the dog limps on the right foreleg as it enters the exam room, the owner reports that the dog was lame on the left hindleg last week. On physical exam, the dog reacts painfully on palpation of the long bones of the right foreleg. Which one of the following choices is the most likely diagnosis? Value Normal 99.8 F (37.7 C) 99.5-102.5 F, 37.2-39.2 C HR=100 bpm 0-120 RR=24 brpm 15-34 A - Panosteitis B - Multiple cartilaginous exostoses C - Hypertrophic osteodystrophy D - Hypertrophic osteopathy E - Retained ulnar cartilage cores

B. Desmopressin acetate The correct answer is desmopressin acetate (DDAVP). Administration of desmopressin results in release of von Willebrand factor, which will help this patient with clotting. Given this dog's breed and elevated BMBT there is a very strong likelihood she is afflicted with von Willebrand's disease. In Dobermans this results in an inability to form a clot. This can be life threatening if the dog is taken to surgery. A whole blood transfusion does not provide an adequate source of von Willebrand factor but may be necessary if the patient's bleeding cannot be controlled despite appropriate pre-operative measures. 1,25 dihydrocholecalciferol is the active form of vitamin D which aids intestinal resorption of calcium. The BMBT does not assess factors 2, 7, 9, or 10 and therefore vitamin K is not indicated.

A 1-year old female spayed Doberman Pinscher has presented after being hit by a car. Initial chest radiographs show mild contusions, and the patient appears to be otherwise stable. A right mid-shaft long oblique femoral fracture has been identified. Routine pre-operative blood work is unremarkable. A buccal mucosal bleeding test (BMBT) is elevated at 6 minutes. What will you administer prior to surgery? A. 1,25 dihydrocholecalciferol B. Desmopressin acetate C. Vitamin K D. Whole blood transfusion

D. Sarcoptes infestation The key to this question is that Sarcoptic mange is extremely pruritic and can be transmitted to people, including this dog's owner. Diagnosis of this disease is usually based on clinical impression and potential for exposure to the mites. A positive skin scraping would be most definitive, but scrapes often come back negative and trial therapy would need to be instituted. Demodex is not transmittable to people and does not typically cause extreme pruritus like Sarcoptes. Lymphoma and squamous cell carcinoma are less likely because they typically would occur in older animals and neither are usually particularly itchy.

A 1-year old mixed breed stray dog is presented for extreme pruritis and the lesion shown in the picture. The dog was recently found and adopted off the street and has been itching despite being bathed and treated with flea preventatives. The owner reports that she has been itching and developing rashes on her own body since adopting this dog. Your physical exam shows that the dog has several additional similar lesions on the other legs, chest, and ventral abdomen. What is the most likely diagnosis? Image used with permission, from Small Animal Dermatology (Moriello), courtesy of Manson Publishing. A. Demodex infection B. Cutaneous lymphoma C. Squamous cell carcinoma D. Sarcoptes infestation

A. IV fluids with added sodium bicarbonate This calf is typical of those suffering from nonspecific calf diarrhea, most often associated with enteropathogenic E coli, rotavirus, or cryptosporidium. The calf develops hypovolemia and metabolic acidosis and requires sodium-containing IV fluids which contain additional alkali such as sodium bicarbonate. One can assess that the calf is severely acidotic given that it is lethargic and unable to stand. While oral fluids may also be useful, at this stage, the calf will require IV fluids.

A 10-day old commercial dairy calf has diarrhea that is white in color (see photo). The calf is dehydrated, hypovolemic, weak and unable to stand. T=100F (37.8 C), HR=100, and RR=20. No other abnormalities are found on physical examination. Based on these findings, what is the treatment of choice? A. IV fluids with added sodium bicarbonate B. Oral fluids containing sodium bicarbonate C. IV fluids containing 50 meq/L of potassium D. Oral fluids containing high levels of both sodium and chloride E. IV fluids containing only saline

E. Hemangiosarcoma The correct answer is hemangiosarcoma. Hemangiosarcoma of the heart has a predilection for the right auricle of dogs, with Golden Retrievers being predisposed. Stabilization of this patient will require pericardiocentesis; prognosis is guarded and may include chemotherapy, pericardiectomy, or rarely auriculectomy. Chemodectoma arises from the ascending aorta and heart base; lymphoma is rare in the heart of dogs and is more commonly metastatic than a single mass. Idiopathic pericardial hemorrhage is incorrect because of the clear evidence of a mass.

A 11-year old male castrated Golden Retriever presents for collapse with muffled heart sounds on examination. You ultrasound the heart and obtain the following image; the right atrium (RA) and right ventricle (RV) are labeled. A large mass is seen in the right atrioventricular groove with pericardial effusion. What is the likely diagnosis? A. Idiopathic pericardial hemorrhage B. Pulmonary adenocarcinoma C. Lymphoma D. Chemodectoma E. Hemangiosarcoma

A. Magnetic resonance imaging (MRI) of the head This cat has the signs and symptoms of acromegaly. Acromegaly is caused by excessive growth hormone release from the pars distalis from a tumor in the pituitary gland. Excessive growth hormone causes a defect in the insulin receptors on target cells causing insulin resistant diabetes mellitus. The enlarged head, paws, abdomen, and weight gain despite uncontrolled diabetes is due to the anabolic effects of the growth hormone. Treatment for this condition includes radiation therapy to the pituitary tumor, high doses of insulin to try and control the diabetes, and somatostatin analogs (octreotide) to try and inhibit the release of growth hormone from the tumor. Surgical excision has been used as a form of treatment in people with pituitary tumors, but this has only been rarely reported in cats.

A 12-year old male neutered domestic short hair cat presents for ongoing evaluation of diabetes mellitus. The cat was diagnosed 6 months ago and has continued to be markedly polyuric, polydipsic, polyphagic, and has been gaining weight. The cat is currently receiving 10 units of glargine insulin every 12 hours. On physical exam, the cat weighs 15 pounds (6.8 kg) and has an enlarged head, abdomen, and paws. What imaging modality would be most appropriate to try and prove what you suspect is causing the uncontrolled diabetes and weight gain in this cat? Image courtesy of Mark E. Peterson, DVM, Dip. ACVIM Animal Endocrine Clinic A. Magnetic resonance imaging (MRI) of the head B. Computed tomography (CT scan) of the abdomen C. Radiographs of the thorax D. Ultrasound of the neck E. Ultrasound of the abdomen

B - Salmonellosis If you see "pseudo-diphtheritic membranes" in the intestines, think Neonatal salmonella. Look also for fever, diarrhea, especially with a "Septic tank odor" and systemic signs like septicemia, pneumonia, meningitis. Can die suddenly without diarrhea. Other rule outs for OLDER calf diarrheas include: Coccidiosis ( Older than 21 days and in ALL ages, bloody diarrhea, tenesmus) Ostertagiasis (anorexia, poor growth, diarrhea in less than 2 year olds on pasture- a chronic disease) BVD (Young cattle 6-24 mos, with mucosal lesions, diarrhea) Winter dysentery (explosive outbreak watery diarrhea with clotted dark blood, stabled animals, all ages in winter) and Johne's disease (Intractable diarrhea, cachexia). Crypto is typically a disease of the young, 1-4 weeks old.

A 2-month old calf is presented that was found dead. The night before this calf was weak, feverish and had a mucoid brown diarrhea with a small amount of blood and a nasty odor. Necropsy reveals a pseudo-diptheritic membrane lining parts of the distal small bowel and large bowel. What is the diagnosis? A - Cryptosporidiosis B - Salmonellosis C - Bovine Viral Diarrhea, mucosal form D - Coccidiosis E - Ostertagiasis

D. Feline infectious peritonitis This poor cat is infected with the dreaded wet form of feline infectious peritonitis. Good job if you nailed it! If you think about the pathogenesis of the virus, then these clinical signs make a lot of sense. With FIP, the classical lesion is pyogranulomatous vasculitis due to antigen-antibody complexes depositing in the venular endothelium, which results in pleural and peritoneal effusion. The CBC findings are common for FIP but not too specific. In greater than 50% of cats with the wet form of FIP, there will be hyperproteinemia. In greater than 70% of cats with the dry form of FIP there is hyperproteinemia.

A 2-year old female DSH cat presents for weight loss, anorexia, dyspnea, and lethargy. She was previously treated with antibiotics but is still febrile on physical exam. You detect pleural effusion and notice that the abdomen is distended. On CBC there is a non-regenerative anemia, neutrophilia, and lymphopenia. On chemistry there is hyperproteinemia and a slight elevation in liver enzymes. What is your primary differential? A. Feline leukemia virus B. Feline calicivirus C. Feline immunodeficiency virus D. Feline infectious peritonitis

B - Deworm cows post-calving, calves by midsummer, all stock in fall Treatment includes deworming cows post-calving, calves by midsummer, all stock in fall. An abomasum with a cobblestone or "Moroccan leather" appearance is pathognomonic for Ostertagia , one of the three stomach worms of cattle. Follow this link to see a Merck image of Ostertagia in an abomasum. These worms remain dormant in the gastric glands of the abomasum, emerging in the spring. Rx is based on regular deworming of whole herd (2-4 times/year). Ivermectin or high dose Albendazole/Fenbendazole, given repeatedly can help treat the dormant ("hypobiotic") Ostertagia in the gastric glands. Amprolium is your treatment of choices for calves and cows with Coccidiosis.

A 2-year old heifer is presented who died the night before. She was stunted compared to her herdmates, and had a 3-week history of poor appetite, unthriftiness, diarrhea. Necropsy reveals that the abomasum is edematous and covered in small umbilicated nodules 1-2 mm in diameter (cobblestone or "Moroccan leather" appearance). Several other younger cows in this Spring-calving herd look unthrifty and have diarrhea too. What is the plan of action? A - Cull the sick cows, Keep horses off same pasture 3 months B - Deworm cows post-calving, calves by midsummer, all stock in fall C - Treat affected heifers with Albendazole D - Treat affected heifers with Ivermectin, repeat in 6 months E - Treat herd with Amprolium, repeat q 5 weeks

C. Methocarbamol Pyrethrins alter the activity of the sodium ion channels of nerves, which prolongs the period of sodium conductance. This increases the length of depolarization resulting in repetitive nerve firing. Cats are particularly sensitive to pyrethrin-containing products and can develop clinical signs within hours after administration. Affected animals should be bathed to remove remaining product. Minor clinical signs such as hypersalivation and ear twitching are usually self-limiting and do not require treatment. Control of marked tremors or seizures can be achieved with methocarbamol (Robaxin).

A 2-year old male neutered cat presents to you depressed, hypersalivating, and ataxic with muscle tremors. The owner reports that a pyrethrin-based spot-on formulation for flea control belonging to their Golden Retriever was accidentally applied on the cat earlier today. Which of the following drugs will you use to treat the cat's clinical signs? A. Atropine B. Amoxicillin C. Methocarbamol D. 2-PAM E. Acepromazine

E. Purpura hemorrhagica This is the clinical and histologic appearance of purpura hemorrhagica. It is a type-III hypersensitivity which is when antigen-antibody complexes accumulate, leading to disease. Purpura hemorrhagica most commonly occurs 2-4 weeks after exposure to certain infectious agents or vaccines. This condition is most commonly seen subsequent to infection with Streptococcus equi subsp. equi or vaccination against it but it can also be associated with other pathogens, particularly respiratory pathogens including other streptococcal species and equine influenza. Regardless of the cause, purpura hemorrhagica results from accumulation of antigen-antibody complexes that deposit on blood vessel walls and activate a strong immune response (vasculitis). The leaky blood vessels lead to hemorrhage and edema. Bastard strangles refers to the condition when Streptococcus equi subsp. equi creates abscesses in unusual sites (other than the lymph nodes draining the throat) such as abdominal or lung lymph nodes. Cantharidin toxicity (also known as blister beetle toxicity) leads to mucosal irritation and results in colic and cystitis. It can also lead to hypocalcemia. Primary immune-mediated thrombocytopenia (sometimes called idiopathic thrombocytopenia) can be seen in horses but is not consistent with the necrotizing vasculitis described in the case. Type I hypersensitivity reactions are immediate reactions (such as urticaria) invoked by an antigen or allergen.

A 2-year old mare presents to you several weeks after recovering from a mild upper respiratory infection. She now presents with edema and sloughing of the legs (see image), chest and abdomen as well as mucosal petechial hemorrhages. She is sore and reluctant to move. Biopsy of the skin lesion is consistent with aseptic necrotizing vasculitis. What is the most likely diagnosis? A. Cantharidin toxicity B. Idiopathic thrombocytopenia C. Bastard strangles D. Type I hypersensitivity E. Purpura hemorrhagica

E. Penicillin The diagnosis is Listeriosis. Listeria monocytogenes can effectively be treated in the early stages of the disease with penicillin, ampicillin, or tetracycline. Intramuscular procaine penicillin for example has a withdrawal of 10 days for slaughter (meat) and 48 hours for milk if the animal is lactating. Other choices of approved antimicrobials would also likely be effective since L. monocytogenes is susceptible to most antimicrobials. For a list of approved animal drugs see www.farad.org/vetgram. FARAD is the United States Food Animal Residue Avoidance Databank which gives withdrawal information as well as a list of prohibited drugs. The other drugs listed as choices here are prohibited and cannot be legally used in food animals in the USA.

A 2-year old nonlactating dairy cow in California has a sudden onset of head tilt and drooling. The owner also reports that she appears less active and less interested in feed than normal, and today is circling in one direction in the pen. You examine her and find T=105F (40.6 C), HR=96, and RR=32. There is ptosis, drooped ear, and weakness of the lips on the affected side. You take a lumbosacral spinal tap (see photo), and submit the CSF to your lab. The results show elevated protein and WBCs, with the cell type being mainly monocytes. The lab reports seeing some gram positive bacteria in the monocytes of the CSF. Based on these findings what is the best treatment for this condition? A. Enrofloxacin B. Metoclopramide C. Metronidazole D. Chloramphenicol E. Penicillin

C - Splints This is a case of splints. Splints are periosteal reactions due to strain on the interosseous ligament between the third metacarpal or metatarsal (cannon) bones and either the second or fourth metacarpal/metatarsal (splint) bones. Click here to see anatomic diagrams of splints, courtesy of the University of Missouri extension service. Trauma from concussion, strain from training, or conformation problems contribute to splint formation. Lameness is variable, usually disappearing as lesions become chronic. If suspensory desmitis develops secondarily, prognosis is more guarded. Bucked shins are a painful, acute periostitis on the cranial surface of the cannon bone. Click here to see a radiograph of bucked shins. Epiphysitis (physitis) involves swelling around the long bone growth plates in young horses and can be a component of osteochondrosis. Ringbone is an ostearthritis of the phalanges and pastern. Click here to see a radiograph of high ringbone.

A 2-year-old Thoroughbred stallion has developed a right foreleg swelling in the region of the distal carpus and proximal cannon bone with variable lameness, most pronounced after exercise. A radiograph of the affected area is below. What is the diagnosis? A - Ringbone B - Epiphysitis C - Splints D - Bucked shins E - Suspensory desmitis

D - Osteosarcoma This is a classic image of osteosarcoma, a COMMON, aggressive bone tumor typically found in the appendicular skeleton, especially distal radius. Here is a memory aid for common locations of osteosarecoma - "Away from the elbow, near the knee" 90% have microscopic metastases to lungs by time of Dx, (but less than 10% will have visible thoracic metastasis at time of diagnosis). Usually, does not cross joint (unlike osteomyelitis, which usually DOES cross joint). Look for soft tissue swelling, periosteal proliferation, sunburst periosteal reaction (33%), possible pathologic fractures.

A 3-year old Irish Setter presents with a history of lameness and swelling in the distal forelimb. A radiograph is shown below. What is the presumptive diagnosis? Click here to see image A - Panosteititis B - Hypertrophic osteodystrophy C - Osteomyelitis D - Osteosarcoma E - Osteochondrosis

C. Itraconazole Based on the description of the cytology and clinical symptoms of this cat, the most likely diagnosis is Cryptococcus neoformans. This is a fungal disease that occurs when the organism is inhaled and is disseminated to skin, eyes, CNS, lungs, or other areas. The upper respiratory tract is most often involved and symptoms can include nasal discharge, sneezing, swelling over the nose, and regional lymphadenopathy. If the CNS is involved, seizures can also occur. The disease has been thought to be transmitted most often through infected pigeon droppings. Itraconazole or fluconazole are the anti-fungals of choice for this disease. Doxycycline, Trimethoprim sulfa, and Clavamox are all antibiotics and would not address a fungal infection. Lufenuron is a flea treatment that has had some implications for treatment of dermatophytes due to its ability to inhibit chitin. About 1/3 of the cell wall of a fungus is composed of chitin. This has not been a promising or approved treatment for ringworm and surely would not be an appropriate treatment for Cryptococcus.

A 3-year old female spayed indoor/outdoor domestic short hair presents for congestion and swelling over the bridge of the nose. The swelling over the nose is firm and seems to be subcutaneous. There is also mild enlargement of the mandibular lymph nodes. You perform cytology from a fine needle aspirate of the swelling over the nose and see narrow, budding, thin-walled yeasts surrounded by clear capsules (see picture). You also detected a lesion in the retina on fundic examination. Which of the following treatments would be indicated? A. Amoxicillin-clavulanic acid B. Trimethoprim sulfa C. Itraconazole D. Doxycycline E. Lufenuron

A. Thrombus at the aortic bifurcation (saddle thrombus) Maine Coon cats are predisposed to development of hypertrophic cardiomyopathy at a young age. Consequently, left atrial enlargement predisposes to atrial thrombus formation, and these clots frequently lodge in the arterial supply to the hindlimbs. The trifurcation is where the aorta divides into the two external iliac arteries and the common origin of the internal iliac arteries. Classic findings due to a clot at the aortic trifurcation include posterior paresis/paralysis, hindlimb pain, cyanotic nailbeds, absent femoral pulses, and a firm leg musculature. Other signs of cardiac disease/failure (murmur or pulmonary edema) are often but not always evident at presentation. Neither cord lesions, tetanus, nor toxicities should cause the vascular compromise evident on this cat's physical exam.

A 3-year old, indoor-only, male castrated Maine Coon cat is presented to you for acute-onset of hindlimb pain and paresis. Physical examination reveals a grade III left parasternal heart murmur and minimal movement in the hind legs. Femoral pulses are bilaterally absent, and the hindlimb toes are cold to the touch. The cat is tachypenic, but lung sounds are normal. What is the most likely diagnosis? A. Thrombus at the aortic bifurcation (saddle thrombus) B. T3-L3 intervertebral disc disease C. Chlorpyrifos toxicity D. Tetanus E. Lymphoma of the spinal cord

B - The hamster is not dead. At temperatures under 41°F (5°C), hamsters will enter pseudohibernation. Though not a true hibernation, they are called "permissive hibernators". They may look "dead" to the owner, but they are alive. Cold temperatures may also stimulate hamsters to gather up food.

A client calls in the middle of winter to say her furnace went out the night before and the whole house became very cold. This morning one pet hamster appears to be dead and several others are lethargic. What message should be communicated to the owner? A - The other animals are likely to die B - The hamster is not dead C - Put tetracycline in the water and maintain the ambient temperature D - Need to examine the lethargic animals and increase calorie intake E - Males are more likely to die from cold stress

B. Neonatal Isoerythrolysis Neonatal Isoerythrolysis (NI) would be a possible cause of the icterus noted in the sclera. NI is common in multiparous mares and usually affects foals between 24-72 hours of age. NI results in hemolysis, hyperbilirubinemia, and icterus. A low packed cell volume would further support NI. NI develops because the newborn foal expresses alloantigens on its red blood cells inherited from the sire that the mare does not have. If the mare becomes sensitized to the sire's specific antigen, maternal antibodies are produced and absorbed by the foal soon after birth. This subsequently results in lysis of the red blood cells.

A 36 hour foal born from a multiparous mare presents for weakness, decreased appetite, and tachycardia. On physical exam, you examine the sclera as seen in the photo (sclera is icteric/yellow). Based on the signalment and findings, what is a likely cause? A. Congenital iron toxicity B. Neonatal Isoerythrolysis C. Failure of passive transfer D. Immune mediated thrombocytopenia

D. They are older intact males This is a case of a perineal hernia. Older intact male dogs are most commonly affected. Overrepresented breeds include Boxers, Collies, Kelpies, Pekingese, and Boston terriers. The hernia results from a weakened pelvic diaphragm. It is thought that there may be a hormonal component which results in weakening with time.

A dog presents to your clinic with tenesmus and swelling near the anus as seen in the image. On examination, there is a fluctuant swelling lateral to the anus, and on rectal exam, you note lateral dilatation of the rectum. Which of the following is the most common signalment for dogs presenting with this problem? A. There is no age or sex predilection B. They are younger intact males C. They are older intact females D. They are older intact males E. They are younger intact females

The correct answer is to culture the discharge for bacteria. The most likely diagnosis for this horse is equine strangles. Strangles most commonly affects younger horses (<5 years of age), but can cause disease in any age horse. The etiologic agent of this disease is Streptococcus equi subsp. equi. The diagnostic test of choice to confirm this is bacterial culture. While awaiting culture results, the horse should be separated from any other horses, as strangles is highly contagious to other horses. Antibiotic therapy is controversial and thought to lengthen the course of disease rather than shorten it when given at this stage; also, it may possibly interfere with the natural immunity acquired from natural infection. This is not a reportable disease.

A 4 year old mare has bilateral swelling and drainage of the mandibular lymph nodes. Rectal temperature is 101.8F (38.8 C). Which of the following is the most appropriate plan for this horse? A. Administer corticosteroids B. Culture the discharge for bacteria C. Inform the state veterinarian D. Administer penicillin

C. Adrenal tumor These tumors are very common in middle-aged ferrets and usually secrete estradiol. Common clinical signs are hair loss, enlarged vulva, pruritus, and behavior changes. Diagnosis is made by clinical signs, ultrasound, and measurement of serum estradiol levels. Treatment is adrenalectomy.

A 4-year old female ferret presents with a history of an enlarged vulva and truncal alopecia. What is the most likely diagnosis? A. Insulinoma B. Sertoli cell tumor C. Adrenal tumor D. Hypothyroidism

B. 20cm H2O Pressures above 20cm H20 may result in barotrauma. In an otherwise healthy patient it is not recommended to exceed this pressure. In patients with chronic atelectasis, anesthetists will be much more apprehensive about over ventilating or ventilating the lungs too quickly, as acute expansion can trigger re-expansion pulmonary edema, which may then lead to acute respiratory distress syndrome and death.

A 4-year old female spayed mixed Chihuahua presented to the emergency service at approximately 5am this morning after presumptively being attacked by a coyote. The patient has a flail chest and it is questionable if there is direct communication between the thoracic cavity and the environment (it was difficult to examine the dog due to her fractious nature). Exploration of the wound was performed, and once anesthetized, it was apparent she had a pneumothorax. The patient must be ventilated, as there is no vacuum present in the chest for lung expansion to occur. What pressure should the anesthetist not exceed if manually bagging the patient during anesthesia? A. 8cm H2O B. 20cm H2O C. 12cm H2O D. 24cm H2O

A. Acetylcysteine and S-adenosylmethionine Acetaminophen toxicity in cats usually occurs when owners administer the drug, unaware of its significant potential toxicity in cats. In this case, the cat's clinical signs are most consistent with acetaminophen toxicity based on the Heinz body anemia that is present. Cats can die from oxidative damage and methemoglobinemia within 1-2 days of ingestion. It may also be associated with hepatotoxicity in cats, although this is seen more frequently in dogs. Recall that cats are particularly sensitive to acetaminophen because they have decreased glucuronyl transferase activity which conjugates acetaminophen to glucuronic acid for excretion. As a result, 50-60 mg (a single tablet) may be fatal for a 4-5 kg cat. Treatment should consist of toxin removal if possible by inducing emesis in some cases. As the cat in this case is already vomiting, this may not be necessary. Activated charcoal is controversial and should only be given if ingestion occurred within hours and should be administered very carefully in cats due to the risk of aspiration. The specific antidote is acetylcysteine which binds to some of the reactive metabolites of acetaminophen and increases the availability and synthesis of glutathione. Other treatments may include S-Adenosylmethionine (SAMe) which has hepatoprotective and antioxidant properties. Cimetidine can be given to inhibit the p450 oxidase in the liver and limit formation of toxic metabolites. Ascorbic acid can also be used as an adjunct treatment to bind toxic metabolites. In cats with signs of hypoxemia from severe hemolytic anemia (PCV <20%), a transfusion and further supportive care may be warranted.

A 4-year old male Manx cat presents to you because the owners found an empty, opened pill vial in the bathroom and the cat vomited. On physical exam, you note ptyalism and facial edema. The cat's mucous membranes are pale and slightly icteric. You perform a blood smear and detect Heinz bodies in erythrocytes. The cat's packed cell volume (PCV) is 26% (30-45%). The owners provide you a list of the medications in the medicine cabinet which are acetaminophen (Tylenol), finasteride (Propecia), enalapril (Vasotec), and omeprazole (Prilosec). What treatments should you institute for this cat? A. Acetylcysteine and S-adenosylmethionine B. Emesis and methylene blue C. Activated charcoal and whole blood transfusion D. Prednisone and amoxicillin

D. Treat the dog with vitamin K1 for 6 weeks The bright green vomit confirms the owner's suspicion of rodenticide ingestion because many rat poisons contain a bright green dye. Dogs that ingest these products may have bright green vomit or stool. Brodifacoum is a vitamin K antagonist commonly used in rodenticides. Ingestion of this compound causes hemorrhaging after several days due to a lack of production of new clotting factors. Treatment for this condition requires vitamin K1 administration for 4-6 weeks.

A 4-year old male neutered Pit Bull mix comes in to your clinic because his owner thinks he might have gotten into rodenticide while she was out of the house 2 hours ago but is not sure. You send her to bring back the box of rodenticide and induce emesis by administering subconjunctival apomorphine. Within 5 minutes, the dog vomits the material shown in the image below. You rinse out the conjunctiva and administer oral activated charcoal. The owner returns with a box of rodenticide that says brodifacoum. What should you recommend? A. Treat the dog with vitamin E and selenium B. Hospitalize the dog for 24 hours to monitor and treat potential neurologic signs C. No additional treatment or monitoring is needed D. Treat the dog with vitamin K1 for 6 weeks E. Check serum calcium levels today and once weekly for 6 weeks

A - Granulomatous meningoencephalitis. The lesion in this case localizes to the left cerebellum causing the left hypermetria and a right paradoxical head tilt.

A 4-year-old Rat Terrier is presented with a 3 day history of progressive stumbling and falling. Physical exam reveals a right head tilt, left sided hypermetria, generalized ataxia and vertical nystagmus. Which one of the following choices is at the top of the differential list? A - Granulomatous meningoencephalitis B - Fibrocartilagenous embolism C - Ascending and descending myelomalacia D - Central pontine myelinolysis E - Amyotrophic lateralizing sclerosis

D - Test herd, cull positives, remove calves from dam at birth This is likely to be Mycobacterium avium subspecies paratuberculosis (Johne's disease). NO Rx: Test-and-cull programs offer best chance of attaining a Mycobacterium avium subspecies paratuberculosis-free herd. Calves, kids, or lambs should be birthed in manure-free areas. For dairy cattle, calves should be removed from dam immediately after birth, bottle-fed pasteurized colostrum or colostrum from test-negative dams. They should be segregated as much as possible from adult cows and their manure until more than 1 yr old. Must also remove calves at birth from the dam to prevent. REPORTABLE in many states.

A 5-year old lactating Holstein dairy cow is being evaluated for a 2-month history of watery diarrhea, weight loss, and decline in milk production. On examination, the cow is bright, alert and responsive, but unthrifty and very thin, with a soft intermandibular swelling. What is the most appropriate management of this herd? A - Cull positives, ensure 4 L. colostrum uptake for calves B - Quarantine herd, treat until herd tests negative C - Test herd, treat positive animals, add coccidiostat to ration D - Test herd, cull positives, remove calves from dam at birth E - Test herd, treat positive animals, test and treat replacements

A. Free gas bloat, failure to eructate This is sometimes called Type 1 vagal indigestion, or free gas bloat. It is frequently associated with swollen mediastinal lymph nodes caused by pneumonia. The signals to or from dorsal rumen receptors, which detect gas pressure and open the cardia, are compromised such that eructation does not occur normally and free gas bloat occurs. This in turn causes the calf to feel full and it eats poorly and loses weight. One needs to treat the pneumonia and perhaps create a temporary rumen fistula to allow the escape of gas until eructation returns to normal.

A 6-month old feedlot steer, which entered the feedlot 4 weeks ago, has lost weight and is now showing an enlarged left flank as shown in the image below. On exam, you find the left side of the abdomen to be gas-filled under moderate pressure, and the rumen to be otherwise poorly filled and with poor motility. Based on percussion and auscultation, the animal also appears to have chronic bronchopneumonia. What is the most likely cause of the rumen malfunction? A. Free gas bloat, failure to eructate B. Type 3 vagal indigestion C. Cecal dilation D. Frothy bloat E. Left displaced abomasum

C - Flip over disease This is the clinical picture of flip-over disease, a production-related disease associated with intensive husbandry. The cause is not known but thought to be related to high carbohydrate intake. Ventricular fibrillation may be the cause of sudden death, and a lack of gross pathology is common. Because broilers affected with flip over disease frequently die on their backs, differential diagnosis includes ascites syndrome (waterbelly). Sick birds with ascites syndrome typically show clinical signs like cyanosis, panting and abdomens distended by fluid. Essentially a form of right ventricular heart failure secondary to pulmonary hypertension or more rarely, hepatic failure. Dissecting aneurysm is another cause of sudden death, but animals display severe hemorrhage on necropsy.

A dozen young chickens at a broiler-raising facility are found dead on their backs and sides with no premonitory signs. Necropsy of a dead bird shows good body condition and no particular pathology. What is the most likely clinical diagnosis? A - Ascites syndrome B - Newcastle C - Flip over disease D - Highly pathogenic avian influenza E - Dissecting aneurysm

C. Immune mediated hemolytic anemia The image shows a positive slide agglutination test. The dark patches in the sample are caused by antibodies attached to the surface of the erythrocytes cross linking the cells together causing these clumps. This is highly suggestive of an immune mediated reaction to antigen on the individual's red blood cells. Rouleaux formations are red blood cells stacked together as a result of their natural discoid shape and large surface area causing them to have an affinity for each other. They typically resemble a stack of coins or cookies. Evan's syndrome is an autoimmune condition characterized by an immune attack on erythrocytes, platelets, and occasionally leukocytes as well. You would not be able to diagnose this on a slide agglutination test alone. Vitamin K antagonist toxicity with rodenticide causes a coagulopathy but should not cause hemagglutination.

A 7-year old female spayed English Springer Spaniel presents for difficulty breathing and lethargy. Physical exam shows pale mucous membranes and increased respiratory effort. You collect blood for a complete blood count and a chemistry panel and you place a drop of blood with a drop of saline on a glass slide. After swirling the sample around, you can grossly see what is shown in the picture. What's your diagnosis? Image used with permission, from Veterinary Cytology (Freeman), courtesy of Manson Publishing. (Image is of coagulated blood) A. Evan's syndrome B. Vitamin K antagonist toxicity C. Immune mediated hemolytic anemia D. Rouleaux formations E. Hemolysis

Left-sided congestive heart failure The cardiac silhouette is tall suggestive of left ventricular enlargement and there is a diffuse unstructured interstitial pattern in the caudoventral and cranial lung fields. The pulmonary vasculature is also dilated. Remember that cats in heart failure can have edema distributed in patchy infiltrates throughout the lungs, in contrast to dogs where it is more typically in the perihilar region. Asthma is less likely as no clear bronchial markings are seen and because of the infiltrate. Neoplasia is a consideration, but doesn't explain the cardiac enlargement. Pneumonia could also cause this pattern of infiltrate (although it is more typically cranioventral), but also does not explain the cardiomegaly. There is no evidence of tracheal collapse, which is an exceedingly rare condition in the cat.

A 7-year old male castrated cat presents for respiratory distress with open-mouth breathing. You initially place him in oxygen and obtain thoracic radiographs when he is stabilized. The films are shown here. What is your diagnosis? A. Pneumonia B. Left-sided congestive heart failure C. Neoplasia D. Tracheal collapse E. Feline asthma syndrome

C. Tensilon response test The correct answer is a Tensilon (edrophonium) response test. Hopefully, you were able to identify the mass in the cranial mediastinum on the chest radiograph, as this was one of the keys to this case. This, in conjunction with the dog's other signs, are suggestive that this dog has a thymoma and associated secondary myasthenia gravis. Tensilon (edrophonium) is a rapidly acting anticholinesterase that reverses signs of myasthenia within minutes in most dogs. A chest CT would be a valid test to confirm the presence of the mediastinal mass and might be an appropriate test before surgery but would not bring you closer to a diagnosis if you already have identified the mass. An MRI of the brain would assess a CNS cause of the dog's signs, which are unlikely, given the other findings. Similarly, a myelogram would assess if a spinal cord lesion caused the dog's signs, but the rest of the findings in this case should point you in a different direction.

A 9-year old male Queensland Heeler presents with a four day history of progressive tetraparesis. Physical exam showed him to be weakly ambulatory with support, and with delayed conscious proprioceptive reactions in all 4 feet. As part of your initial workup, you take chest X-rays which are shown below. Which of the following next steps is the most appropriate test to confirm your clinical suspicion about the cause of the dog's signs? A. CT scan of the thorax B. MRI of the brain C. Tensilon response test D. Bronchoalveolar lavage and culture E. Myelogram

B. Constipation Lethal white foals have aganglionosis of the intestines which leads to hypomotility, megacolon, constipation, colic, and death

A Paint horse mare gives birth to an all white foal (see image). What clinical sign would you expect to see in a foal with lethal white syndrome? A. Regurgitation B. Constipation C. Anhydrosis D. Ataxia

A - Polioencephalomalacia The two most common causes of polioencephalomalacia (PEM) are low thiamine (due to thiaminase activity from plants like bracken fern or low thiamine diets) and high sulfur in the diet (which can come from a high molasses-urea diet, corn or sugar cane byproducts, water, or other plants, including alfalfa, Canada thistle (Cirsium arvense), kochia, (Kocchia scoparia), and lambsquarter (Chenopodium spp).

A bovine diet that is low in thiamine or high in sulfur can cause ____________. A - Polioencephalomalacia B - Pregnancy toxemia C - Parturient paresis D - Downer cows E - Pseudorabies

B. BUN and creatinine levels Lily plant toxicosis is extremely serious and can cause rapid and fatal acute renal failure in cats. If ingestion is suspected, decontamination and aggressive fluid therapy, and monitoring of renal values are recommended immediately. In this case, since 2 days have passed, inducing emesis and administering activated charcoal would not be helpful. What you can do is check renal values (BUN and creatinine) and treat for acute renal failure. Ruling out an intestinal obstruction in a cat with this history is reasonable but not the best of the choices given.

A client calls and says her cat was chewing on her lily plant two days ago and is now acting very sick. You tell her to bring the cat in immediately so that you can perform which of the following? A. ALT, AST, GGT, and total bilirubin levels B. BUN and creatinine levels C. Abdominal radiographs to diagnose intestinal obstruction D. Induce emesis and administer activated charcoal

B. Carprofen This is a case of panosteitis based on the young age of the patient and radiographic presence of focal intramedullary densities within the tibial diaphysis. Minor differentials could include osteomyelitis. Panosteitis is a self-limiting, painful condition characterized by limping and lameness. It typically affects the long bones of young dogs, usually between the ages of 5 to 18 months. It can occur with any breed, but it is more common in medium- to large-sized dog breeds. Treatment is primarily supportive consisting of limiting activity and anti-inflammatory drugs. Therefore, carprofen is the best answer choice listed. Pain lasts from weeks to months and resolves in nearly all cases. While these treatments reduce the pain associated with the condition, they may not alter the duration or course of the disease.

A 9-month old male pitbull presents to you for hindlimb lameness that developed suddenly several days ago and has gotten worse. On your examination, the dog is painful on palpation of the left tibia. You take radiographs of the tibia which are shown below (increased densities in the intermedullary canal) . Which of the following is the most appropriate treatment for the suspected condition? A. Ampicillin B. Carprofen C. Fluconazole D. Splint stabilization E. Surgical correction

B - Histomoniasis This is histomoniasis. The combination of characteristic "bulls-eye" lesions on liver and cecal changes are pathognomonic. Caused by protozoan Histomonas meleagridis, transmitted in eggs of cecal nematode Heterakis gallinarum. Expect a depression/diarrhea presentation. Expect to see more sudden death with necrotic enteritis caused by Clostridium perfringens. Follow this link to see the so-called "Turkish towel" intestinal pseudomembrane of necrotic enteritis. Signs of avian spirochetosis are highly variable, may be absent: see listlessness, shivering, increased thirst, green/yellow diarrhea with increased urates early on. Caused by a tick-borne Borrelia. Look for characteristic enlarged, mottled spleen with petechial hemorrhages, similar to Marble spleen disease of pheasants. Expect depression, bloody droppings, substantial mortality with hemorrhagic enteritis of turkeys. Follow this link to see hemorrhagic intestines. Follow this link to see characteristic enlarged spleen. Expect diarrheal presentation with coronaviral enteritis of turkeys but NOT the characteristic cecal/liver lesions described on necropsy above.

A flock from a turkey farm is presented with a mysterious illness. Several dead birds are noted, mostly younger. Sick turkeys are listless, with drooping wings, unkempt feathers, yellow droppings. Sick older birds are emaciated. Necropsy shows a yellowish green, caseous exudate in the ceca, cecal ulcerations and thickening of the cecal wall. A typical liver looks like the image below. What is the diagnosis? A - Necrotic enteritis B - Histomoniasis C - Avian spirochetosis D - Coronaviral enteritis of turkeys E - Hemorrhagic enteritis of turkeys

This is Fowl Cholera, caused by Pasteurella multocida. Think sudden onset septicemia. Signs VARY greatly. In acute fowl cholera, dead birds are first indication of disease. May see clinical picture described above. With chronic cases, see localized infections: Sternal bursae, wattles, joints, tendon sheaths, footpads SWELL with fibrinosuppurative exudate. See torticollis if meninges, middle ear, cranial bones infected. Follow this link to see original Merck image of Fowl Cholera. With Infectious Coryza think acute respiratory disease with nasal discharge, sneezing, and swelling under the eyes, caused by Avibacterium (Haemophilus) paragallinarum. Follow this link to see an image of Infectious Coryza. With Infectious Laryngotracheitis (ILT) look for gasping, coughing, blood stained beaks, blood occluding trachea on necropsy. In most states ILT is REPORTABLE. Follow this link to see an image of ILT. Infectious Bronchitis is characterized by respiratory signs, decreased egg production and poor egg quality. Classically may see "wrinkled eggs" with inf Bronchitis . Newcastle disease is characterized primarily by respiratory signs. Severe forms include depression, neurologic signs or diarrhea. Look for GI hemorrhage with most severe form, Viscerotropic Velogenic Newcastle disease (VVND), which is REPORTABLE!

A flock of chickens from a poultry operation is presented with an outbreak which began 2 days ago with the unexpected discovery of 20 dead birds. Since then, illness has affected about 25% of the flock. Birds are depressed and anorexic, many with mucoid discharge from the beak, ruffled feathers, diarrhea and increased respiratory rate. Two birds have torticollis, and several chickens with swollen sternal bursae, wattles, joints, tendon sheaths, and footpads. Image- What is the clinical diagnosis? A - Infectious Bronchitis B - Fowl Cholera C - Infectious Coryza D - Infectious Laryngotracheitis E - Newcastle disease

C. Creatine kinase (CK) K, also called, CPK, is found in muscle cells and is elevated in the serum when muscle damage occurs.

A horse is suspected of having the muscle disease rhabdomyolysis, and you are seeking a laboratory test to help confirm the diagnosis. Of the following, which test would likely be the most helpful? A. Ionized phosphate B. Gamma glutamyl transferase (GGT) C. Creatine kinase (CK) D. Anion gap E. Sorbitol dehydrogenase (SDH)

B. Cushing's disease (Pituitary Pars Intermedia Dysfunction) The correct answer is Cushing's disease. The coat condition described is what horses with glucocorticoid excess develop; it is referred to as hirsutism. They will also be predisposed to infections including laminitis and skin diseases such as Dermatophilus. They are also frequently polyuric, polydipsic, and polyphagic.

A horse presents to you for chronic, recurrent laminitis and skin disease. You notice on your exam that the horse has a particularly thick, long, wavy, and matted coat. The owner mentioned that this developed many months ago. What is a likely diagnosis? A. Pheochromocytoma B. Cushing's disease (Pituitary Pars Intermedia Dysfunction) C. Diabetes insipidus D. Hypothyroidism

C. Urinary tract obstruction The correct answer is urinary tract obstruction due to calculi. This should be suspected in all male and castrated male goats (and sheep) with non-specific signs of disease or discomfort because it is so common. Common clinical signs associated with urinary tract obstruction are vocalization and dribbling of urine. Heaving or forceful abdominal contractions may be seen. Hematuria, dysuria, prolonged urination, and apparent abdominal pain are also common signs. The most important step of evaluation is exteriorization of the penis and examination of the urethral process because this is the most common site of blockage. In severe cases the entire urethra may be filled with calculi.

A male goat presents for vocalizing and straining to urinate. He has been kicking at his abdomen. You detect crystals adherent to the hairs around the prepuce (see image). What condition should you suspect and try to rule out first? Image courtesy of David Van Metre A. Upper intestinal obstruction B. Acute severe pyelonephritis C. Urinary tract obstruction D. Lower intestinal obstruction

It is 48%. The trick with this kind of question is to pick an imaginary number of animals that you test, like 1000, and fill out your 2x2 table from there. Follow the links to see diagrams step by step. If prev is 5% then there must be 50/1000 cats with FeLV and 950 cats that are disease-free. A 90% sensitive test will correctly call 45/50 positive (box "a"), and IN-correctly call 5/50 negative, (box "c": these are the false negs). If 50/1000 animals are infected, then 950/1000 are disease-free. Your 95% specific test will correctly call 902/950 disease-free (box "d": 0.95 X 950=902) and IN-correctly call 48/950 positive, (box "b": these are the false pos). Now your a,b,c,d boxes are all filled, it is easy to calculate PVP =a/(a+b)=45/(45+48)=48%

A practice is using an FeLV test with a sensitivity of 90% and a specificity of 95%. Assuming the prevalence of feline leukemia in the area is 5%, what is the predictive value positive (PVP) of the test? A - 45% B - 48% C - 55% D - 88% E - 90%

A. The thin, white, glistening amniotic membrane emerges from the vulva This case description is consistent with stage I of labor in the horse. The first stage of foaling typically lasts 30 minutes to 4 hours. During this stage, mares act restless and may exhibit signs similar to colic such as flank watching, pawing, and constantly getting up and down. When the placenta ruptures ("water breaks"), there may be several gallons of allantoic fluid that come out. Usually, within about 5 minutes, the second stage of labor begins and the foals feet and nose appear at the vulva, covered in the white, thin, glistening amnion. If a red, velvety, membrane is seen, this is the chorioallantois which indicates premature placental separation which can impair oxygen delivery to the fetus and can result in death of the foal. Usually, the muzzle will emerge from the amnion by the time the foal's hips pass through the pelvis but if not, the amnion can be gently broken and removed. Usually, the umbilical cord breaks naturally when the mare stands or foal begins to rise. Then, within 30 minutes to 3 hours after foaling, the placenta should be expelled.

A pregnant mare was brought out to your barn for observation in anticipation of parturition. After several hours of restless behavior, several gallons of allantoic fluid rush out from the vulva. Which of the following would you expect to happen next for a normal parturition? A. The thin, white, glistening amniotic membrane emerges from the vulva B. The hind legs of the foal emerge from the vulva C. The red, velvety, chorioallantoic membrane emerges from the vulva D. The placenta is expelled from the vulva

A - Treponematosis, Hutch burn Treponematosis (rabbit syphillis, vent disease) and hutch burn (urine burn) resemble each other and are often confused. Treponematosis (rabbit syphillis, vent disease) is a venereal disease of rabbits caused by Treponema paraluis cuniculi. Affects the genitalia, may affect eyes and nose. Click here to see a rabbit with cutaneous treponematosis. Hutch burn is caused by wet and dirty floors, affecting the anus and genitalia. Click here to see a rabbit with hutch burn and here to see severe dermatitis secondary to hutch burn. Remember that cauda equina neuritis (polyneuritis equi) in horses may present with urine scald on the thighs. Other signs include a weak tail, hypotonic anus, urine dribbling and fecal retention. There may be a history of rubbing or chewing the tail head.

A rabbit is presented with inflamed and chapped membranes of the anus and genital region. The genital area is scalded and raw, with brownish crusts and purulent exudate. What two conditions top the differential diagnosis list? A - Treponematosis, Hutch burn B - Tularemia, Cystitis C - Myxomatosis, Moist dermatitis D - Glomerulonephritis, Coccidiosis E - Pasteurellosis, Ulcerative pododermatitis

B - Anticoagulant rodenticide toxicity Anticoagulant rodenticide toxicity (or liver disease) can demonstrate increased activated partial thromboplastin time (aPTT) and prothrombin time (PT).

A three year old male neutered Weimaraner dog is presented with a two day history of worsening anorexia, lethargy, cough and exercise intolerance. On physical exam, the mucous membranes are pale with a few petechiae. T=99.8 F (37.7 C)..[N=99.5-102.5 F, N=37.2-39.2 C] RR=24 brpm.....[N= 15-34] HR=144 bpm....[N= 60-120] A coagulation profile shows the following: Thrombocytes= 343,500 per microliter..[N=200,000-900,000] Buccal mucosal bleeding time (BMBT), normal Activated partial thromboplastin time (aPTT), increased Prothrombin time (PT), increased Thrombin time (TT), normal Fibrin degradation products (FDPs), normal Which one of the following choices is the most appropriate diagnosis? A - Disseminated intravascular coagulation (DIC) B - Anticoagulant rodenticide toxicity C - Von Willebrand's disease D - Idiopathic thrombocytopenia E - Neonatal isoerythrolysis

D - LH monitoring Daily testing for the LH surge is the most accurate method of ovulation timing. The day of the LH surge becomes "day 0" and you plan breedings from there, typically at days 2, 4 and 6. You might choose LH monitoring for frozen or chilled semen breedings, infertility cases or breedings with valuable, or subfertile studs. Progesterone assays, vaginal cytology and vaginoscopy are all useful adjuncts to breeding management. Progesterone starts to increase at about the same time as the LH surge- although progestrone levels are not as accurate as measuring the actual LH surge, the tests may be more convenient and available than LH tests. Estrogen measurement is of little value for ovulation timing because peak levels vary from dog to dog and do not necessarily correlate with the fertile period. Increased estrogen speeds up the turnover of vaginal epithelial cells, which causes progressive cornification seen on vaginal cytology.

A valuable female show dog is approaching estrus and the owner wants her bred by artificial insemination to one of the most successful and expensive stud dogs in the country. What testing method is most accurate to insure optimum timing for breeding the bitch? A - Estrogen levels B - Progesterone assays C - Vaginal cytology D - LH monitoring E - Vaginoscopy

A - Vitamin E deficiency Vitamin E deficiency. Acquired equine motor neuron disease uncommon today due to horse owner awareness. When it occurs, it is usually seen in older adult horses. Owners may say that the horse can "walk better than it can stand" when a horse has acquired equine motor neuron disease. The horse will stand still trembling in a tucked up stance, but it will walk well (but in short strides).

Acquired equine motor neuron disease is a disease characterized by weight loss, paresis with trembling, preference to lying down, and decreased activity. Which one of the following choices is associated with this disease? A - Vitamin E deficiency B - Thiamine deficiency C - Lead toxicosis D - Organophosphate toxicosis E - Rickettsial infection

A - Report case to state health department Suspect chronic wasting disease (CWD) in any adult cervid with aspiration pneumonia. Reportable in most states and provinces, but especially in important in Colorado. CWD was first identified in northern Colorado in 1967 and remains more highly endemic there than anywhere else in the U.S. Click here to see a USDA map of CWD distribution in wild cervids the US (scroll down to page 3). Although weight loss is progressive throughout disease, remember that CWD may be present in deer and elk that are not thin or emaciated. Some animals may die of CWD without gross lesions. Death or clinical illness in CWD-affected animals can be precipitated by stressors like cold weather or restraint. When present, the most common gross lesions due to CWD are poor body condition, watery rumen contents, and dilute urine.

After a stressful period of cold winter weather in northern Colorado, a petting zoo reports that one adult mule deer out of a herd of eight is sick. On physical exam, the animal has a fever, increased respiratory rate and pulmonary rales, suggesting aspiration pneumonia. Which one of the following choices is the most appropriate next step to take? A - Report case to state health department B - Treat sick animal with high-dose penicillin C - Cull sick animal, vaccinate herd against histophilosis D - Treat herd with high-dose penicillin E - Perform an intradermal tuberculin test

D - High Mountain disease Bilateral dilated jugulars says "Heart problem". Hx of high altitude exposure and brisket / submandibular edema point towards High Mountain disease. Remember Locoweed (Astragalus and Oxytropis) ingestion worsens disease, due to swainsonine.

An 18 month old steer is presented that is just down from summer pasture in the mountains of Colorado with brisket edema, slight dyspnea and distended jugular veins. What condition is at the top of the differential diagnosis list? A - Hardware disease B - Cardiac lymphosarcoma C - White muscle disease D - High Mountain disease E - Cor pulmonale

C. Class II This is the clinical picture of class II heartworm disease (ie: moderate disease). In a nutshell, Class I is mild and class III is severe. Class IV is an acute shock-like syndrome. Canine heartworm disease can be classified with 4 parameters--physical exam, thoracic radiograph, packed cell volume (PCV) and urinalysis. Class I heartworm disease is an asymptomatic-to-mild case with no radiographic signs and no lab abnormalities. Class II heartworm disease is moderate, with occasional coughing and mild-to-moderate exercise intolerance. On physical exam there may be slight loss of body condition and increased lung sounds. Mild-to-moderate radiographic changes, like right ventricular enlargement are present. Lab results may show anemia and proteinuria. Class III heartworm disease is severe. Variably characterized by weight loss, exercise intolerance and tachypnea at rest. Look for severe or persistent coughing, dyspnea, hemoptysis, fainting and ascites. Severely abnormal radiographs may show right ventricular hypertrophy, enlargement of the main pulmonary artery, and diffuse pulmonary densities. Labwork will indicate marked anemia, thrombocytopenia, and proteinuria. Class IV heartworm disease (also called post-caval or caval syndrome) is an acute, shock-like presentation caused by retrograde worm migration from the pulmonary artery through the right ventricle, right AV valve and into the vena cava. Caval syndrome is characterized by acute collapse, hemoglobinuria, and respiratory distress. Usually fatal if surgery is not immediately instituted to physically remove worms.

An 8-year old dog is presented with a 2-week history of occasional cough and moderate exercise intolerance. On physical exam there are increased lung sounds. Thoracic radiographs show right ventricular enlargement. Lab testing shows that the dog is heartworm antigen-positive, anemic and has proteinuria on urinalysis. What class of heartworm disease is this? A - Class III B - Class I C - Class II D - Class IV E - Cannot say without more information

This is the clinical picture of pericardial effusion. Note the spherical "balloon heart." RX of choice is pericardiocentesis. Ascites, distended jugulars suggests RIGHT congestive heart failure (CHF), more than left CHF (pulmonary edema, crackles, wheezes, cough). Cor pulmonale is just another name for right CHF due to pulmonary hypertension. Dilated cardiomyopathy (DCM) should be on your DDX. More likely to hear systolic murmur, arrhytmias, less likely to be muffled heart sounds. MECHOCARDIOGRAPHY resolves DDX between pericardial effusion, DCM and peritoneopericardial hernia.

An 8-year-old German shepherd is presented with a 1-month history of cough, labored breathing and lethargy. He has fainted a few times after exertion. Upon physical exam, pallor and a slow capillary refill time is noted. The dog has jugular distension, diminished, quiet heart sounds, ascites and weak, variable femoral pulses. A thoracic radiograph reveals the following. What is the clinical diagnosis? Image- thoracic rad with extremely large balloon shaped heart Value Normal T=102.0 F (38.9 C) 99.5-102.5 F, 37.2-39.2 C HR=140 bpm 60-120 RR=24 brpm 15-34 A - Peritoneopericardial hernia B - Dilated cardiomyopathy C - Congestive heart failure (left) D - Pericardial effusion E - Cor pulmonale

B - Rectal palpation in 1st 42 days of pregnancy Vascular damage secondary to amniotic vesicle palpation in the first 6 weeks of embryonic development can lead to intestinal ischemia and atresia in calves. Inherited atresias of the intestine are relatively common in large animals. Only 30% of calves with atresia coli survive to adulthood. Surgical correction is not recommended in Holstein calves because atresia coli is probably heritable in this breed. Click here to see a barium series radiograph of atresia coli in a calf and a necropsy image from same calf. Congenital atresia ani, (imperforate anus, seen most in cattle, sheep, pigs) occurs when the dorsal membrane separating the rectum and anus fails to rupture. Clinical signs at birth include tenesmus, abdominal pain and distention, retention of feces, absence of an anal opening

Atresia coli in the calf is associated with which one of the following choices? A - Freemartins B - Rectal palpation in 1st 42 days of pregnancy C - History of ketosis/hepatic lipidosis in dam D - Oral doxycycline E - Dry matter intake over 20% during dam's peak lactation

A - Retained eggs Egg retention in chelonians is nearly impossible to detect without radiographs. Even WITH radiographs, deciding whether the eggs are ok or retained is difficult. Animals may LACK of clinical signs and you may not have reliable information on breeding dates. If eggs are abnormal (irregular shapes, variable sizes or shell thickness) or, if the turtle shows clinical signs such as straining or cloacal swelling +/- depression or respiratory distress, you should remove the eggs. Diagnosis - Physical examination can reveal the presence of masses and radiography is essential to distinguish whether there are retained eggs, uroliths, or tumors. Medical Rx - Improve husbandry, correct metabolic abnormalities, administer oxytocin or prostaglancin. Surgical Rx - Ovariosalpingectomy is often necessary.

An adult tortoise is presented with a recent history of cloacal straining and depression. Gentle manipulation/rocking of the animal allows palpation of a firm, coelomic structure. Radiography reveals the following image (below): What is the presumptive diagnosis? Click here to see image A - Retained eggs B - Plastron fracture C - Abdominal neoplasia D - Fecaliths E - Uroliths

E - Edema disease Edema disease is caused by Shiga toxin-producing E. coli (STEC). Look for severe acute illness ranging from peracute death with no signs to CNS involvement with ataxia, paralysis, and recumbency in healthiest pigs 1-2 weeks after weaning. Hemagglutinating encephalomyelitis virus (HEV), is almost exclusive to piglets less than 4 weeks old. Two clinical presentations: Vomiting and wasting disease (VWD) and encephalitic. Clostridium perfringens type C enteritis ,also called enterotoxemia in other animals, is characterized by a HEMORRHAGIC diarrhea in 1-3 day old piglets. Porcine proliferative enteritis is principally a diarrheal disease of growing finishing (40- to 80-lb) pigs and young breeding pigs. Epidemic transmissible gastroenteritis (TGE) in non-immune pig herds characterized by HIGH MORBIDITY and HIGH MORTALITY in piglets less than 1 week old.

An outbreak of diarrheal disease of piglets has occurred which affected the healthiest animals in the herd, 1-2 weeks after weaning. Some affected piglets had no signs except peracute death. Other affected piglets exhibit diarrhea, ataxia, paralysis, and recumbency. What condition is at the top of the differential diagnosis list? A - Clostridium perfringens type C enteritis B - Hemagglutinating encephalomyelitis virus (HEV) C - Porcine proliferative enteritis D - Epidemic transmissible gastroenteritis (TGE) E - Edema disease

D - Aortic stenosis

During a routine immunization visit for a 2 year-old neutered male Newfoundland dog, a systolic ejection-type (crescendo-decrescendo) murmur is detected, audible loudest on the left side of the chest between the 2nd and 5th intercostal (IC) space and at the thoracic inlet lateral to the trachea. Which condition is highest on a differential diagnosis list? A - Pulmonic stenosis B - Mitral dysplasia C - Tricuspid dysplasia D - Aortic stenosis E - Patent ductus arteriosus (PDA)

E. Melanoma The correct answer is melanoma. Melanoma is one of the most common tumors in horses (about 10% of all neoplasms) and gray horses are at high risk of around 80%. They can occur anywhere but appear most frequently in the perineal region or ventral tail as is evident in this horse. In horses, they are usually darkly pigmented (as opposed to dogs where amelanotic melanomas occur somewhat commonly). In horses, most are slowly growing but can be locally invasive. Many treatments are out there, but there is no standard of care treatment. Depending on location and extent, consider surgical removal, benign neglect, chemotherapy (systemic or intralesional) and immunotherapy.

During your examination of an 18-year old horse, you observe what is shown in the photograph (black bumps on underside of tail) . Based on the location and appearance of this lesion, what is the most likely diagnosis? A. Cuterebra B. Squamous cell carcinoma C. Sarcoid D. Habronema E. Melanoma

D - Alterations in diet. Mild, self-limiting diarrhea in a foal 7-14 days of age is called foal heat diarrhea, because it coincides with the first estrus cycle post-foaling in the dam. Causes of foal heat diarrhea are poorly understood, but are thought to be related to a foal's tendency to start sampling hay and grain and practice coprophagy by 5 to 7 days of age, with consequent alterations in bacterial flora. Foals are active and alert, with a normal appetite and vital signs. Clinical signs such as fever and lethargy, hematochezia or melena, and laboratory findings such as neutropenia are not routinely observed.

Foal heat diarrhea is typically associated with which choice? A - Hemorrhagic enteritis B - Neutropenia and fever C - Decreased suckling D - Alterations in diet E - Warm and humid weather

B. ACTH Stimulation. The two diagnostic screening tests used most commonly for diagnosis of hyperadrenocorticism are the ACTH stimulation test and the LDDS. Remember your Ps and Ns. ACTH Stim is the most sPecific (fewer false Pos, so trust a POS test more). Click here for a diagram LDDS is more seNsitive (fewer false Negs, so trust NEG test more). Click here for a diagram Either way, these 2 tests are useful to screen for Cushing's, but a confirmatory test is usually required if you get a positive, to differentiate between a pituitary HAC or an adrenal HAC case [high-dose dexamethasone suppression test (HDDST) or ACTH concentration]. The Urine Cortisol Creatinine Ratio (UCCR) has a low specificity (~25%), which means high false positives. 75% of dogs with non-adrenal illness will have a UCCR result consistent with HAC. So, a positive UCCR is not useful to identify HAC, however, a negative is very useful to rule-out HAC as dogs with a normal UCCR cannot have HAC.

Hyperadrenocorticism is suspected in a 9-year old female spayed dog with a two-month history of increased appetite, thirst and urinary accidents. Which of the following diagnostic test is most reliable when results are positive? Urine Cortisol Creatinine Ratio (UCCR). Sensitivity=90% , Specificity =25% ACTH Stimulation. Sensitivity=80% , Specificity =85% Low Dose Dexamethasone Suppression. Sensitivity=95% , Specificity =50% A - Low Dose Dexamethasone Suppression (LDDS) B - ACTH Stimulation C - Urine Cortisol Creatinine Ratio D - Cannot say without knowing the positive predictive value E - Cannot say without knowing the negative predictive value

E - Horses This is rouleau formation, a normal finding in many horses. (Note also the characteristic equine eosinophil in the center) For normal goats, think of marked poikilocytosis. Remember Camelids (llamas, camels, alpaca) have ellipsoid red blood cells (RBCs). Remember that it is normal for birds and reptiles to have nucleated RBCs.

In which animal is this a normal finding? Eosinophil with large granules and RBC's in rouleax formation A - Ferrets B - Goats C - Pot belly pigs D - Calves less than 4 weeks old E - Horses

A - Holstein-Friesian Holstein-Friesian cattle inherit syndactyly, or "mule foot", a simple autosomal recessive trait, more often than other breeds. Syndactyly is the partial or complete fusion of the digits of one or more feet. Forefeet are affected most often. Affected animals walk slowly, usually with a high-stepping gait. They may also be more prone to hyperthermia than normal cows.

In which one of the following cattle breeds is syndactyly most commonly inherited? A - Holstein-Friesian B - Angus C - Hereford D - Brown Swiss E - Simmental

B. Mosquito control The condition described here is is the dry form of avian (fowl) pox. This is a relatively slow spreading disease that can be spread by contact or by mosquitoes that may harbor infective virus for greater than a month. In the dry form of the disease, the main sign is raised, whitish wart-like lesions on unfeathered areas (head, legs, vent, etc.). The lesions heal in about 2 weeks. Unthriftiness, decreased egg production and retarded growth may be seen. Mortality is low with this form of the disease. The wet form mainly involves the oral cavity and upper respiratory tract. Lesions are diphtheritic and can ulcerate or erode mucous membranes. Marked respiratory involvement can lead to mortality A diagnosis is usually based on flock history and presence of these lesions. This is a pox virus and there is no specific effective treatment but there is a vaccine. Disease control is best accomplished by preventive vaccine as sanitation alone will not prevent spread of disease. Several vaccines are available and a single application results in permanent immunity. There are not many tick borne poultry diseases but they may include spirochaetosis and Pasteurella infection. Raising the temperature 5 degrees may be part of the treatment for infectious bronchitis in chickens. Disinfecting pens +/- quarantine is done for quail bronchitis, aspergillosis, and ulcerative enteritis. Antibiotics in the drinking water are most effective for preventing secondary bacterial infections and for mycoplasma but not preventing spread of the virus.

Many turkeys on a poultry farm develop whitish "wart-like" nodules and scabs on the comb, wattles, feet, and vent. Which management intervention would help prevent spread of the disease? A. Add antibiotics to the drinking water B. Mosquito control C. Raise the room temperature 5 degrees D. Immediate removal of fecal waste E. Thoroughly disinfect pens and equipment F. Tick control

C - Segregated early weaning 12-18 mos after initial event 12-18 months after the initial outbreak, sows are usually no longer transmitting the porcine reproductive and respiratory syndrome (PRRS) virus. So, segregated early weaning and removal of nursery pigs (with concurrent disinfection of nursery before repopulation) may help control the virus. Strains of PRRS virus vary markedly in virulence, and there is genetic and antigenic variability between isolates. Recovered sows are resistant to re-infection when exposed to a homologous (same/similar) strain of the virus but are likely to show clinical signs if a heterologous (new/different) strain is encountered. Because it appears that sows generally only abort once due to a particular PRRSV strain, culling after the first farrowing not advisable. Antibiotic administration can help control secondary bacterial infections, but not the arterivirus responsible for PRRS. Adding negative gilts to a herd already infected with the virus is unlikely to result in control. There is no single successful strategy for control of PRRS. You can try to eliminate PRRS (difficult) or try to live with PRRS, which is called breeding herd stabilization. The goal of many farms is to live with PRRSV infection in the sow herd by acclimatizing them to a particular strain of the virus. Herd immunity to that specific strain of PRRSV (eventually) prevents reproductive failure and can decrease transmission of virus from sows to fetuses and offspring. This is stabilization. Acclimatization to a herd PRRSV strain minimizes clinical PRRS, especially when combined with segregated rearing of piglets. Ohter breeding herd stabilization interventions include by vaccination, intentional whole-herd infection, aggressive acclimatization of replacement breeding stock to the herd PRRSV strain, or combinations of these.

Porcine reproductive and respiratory syndrome (PRRS) is characterized by postweaning respiratory disease in piglets and by reproductive failure in sows (ie: increased stillborn and weak-born piglets, mummified fetuses, premature farrowings). Which choice is the most effective control measure in infected herds after an outbreak of a new strain of PRRS virus? A - Cull all sows after first farrowing B - Maintain therapeutic levels of streptomycin in feed C - Segregated early weaning 12-18 mos after initial event D - Buy PRRS-negative gilts, test on arrival, segregate 45 days, retest before join herd E - Vaccinate boars in contact with sows, maximize pen ventilation

C - Segregated early weaning 12-18 mos after initial event 12-18 months after the initial outbreak, sows are usually no longer transmitting the porcine reproductive and respiratory syndrome (PRRS) virus. So, segregated early weaning and removal of nursery pigs (with concurrent disinfection of nursery before repopulation) may help control the virus. Strains of PRRS virus vary markedly in virulence, and there is genetic and antigenic variability between isolates. Recovered sows are resistant to re-infection when exposed to a homologous (same/similar) strain of the virus but are likely to show clinical signs if a heterologous (new/different) strain is encountered. Because it appears that sows generally only abort once due to a particular PRRSV strain, culling after the first farrowing not advisable. Antibiotic administration can help control secondary bacterial infections, but not the arterivirus responsible for PRRS. Adding negative gilts to a herd already infected with the virus is unlikely to result in control. There is no single successful strategy for control of PRRS. You can try to eliminate PRRS (difficult) or try to live with PRRS, which is called breeding herd stabilization. The goal of many farms is to live with PRRSV infection in the sow herd by acclimatizing them to a particular strain of the virus. Herd immunity to that specific strain of PRRSV (eventually) prevents reproductive failure and can decrease transmission of virus from sows to fetuses and offspring. This is stabilization. Acclimatization to a herd PRRSV strain minimizes clinical PRRS, especially when combined with segregated rearing of piglets. Ohter breeding herd stabilization interventions include by vaccination, intentional whole-herd infection, aggressive acclimatization of replacement breeding stock to the herd PRRSV strain, or combinations of these.

Porcine reproductive and respiratory syndrome (PRRS) is characterized by postweaning respiratory disease in piglets and by reproductive failure in sows (ie: increased stillborn and weak-born piglets, mummified fetuses, premature farrowings). Which choice is the most effective control measure in infected herds after an outbreak of a new strain of PRRS virus? A - Cull all sows after first farrowing B - Maintain therapeutic levels of streptomycin in feed C - Segregated early weaning 12-18 mos after initial event D - Buy PRRS-negative gilts, test on arrival, segregate 45 days, retest before join herd E - Vaccinate boars in contact with sows, maximize pen ventilation

C. Anterior uveitis The correct answer is anterior uveitis. Rubiosis iridis along with other signs such as aqueous flare, hyphema, hypopyon, keratic precipitates, and decreased intraocular pressure are all suggestive of anterior uveitis which can be caused by a number of infectious, immune-mediated, traumatic, and idiopathic causes.

Rubiosis iridis as seen in this cat is a sign of what process in the eye? Image Courtesy of Brad Holmberg, DVM MS PhD DACVO A. Lens luxation B. Chorioretinitis C. Anterior uveitis D. Iris atrophy E. Glaucoma

E. Photosensitization Photosensitization mainly affects unpigmented skin where photodynamic agents have accumulated making the skin hyper-reactive to UV light. The cause of this photosensitization can be primary, in which case a plant-derived compound (such as hypericum) or chemical which is injected, ingested or topically applied, is the cause. Alternatively, the cause can be secondary due to hepatic damage/failure in which case the liver fails to remove ingested chlorophyll-breakdown products like phylloerythrin, which accumulates in the skin and results in UV damage (sunburn). You need to determine whether this is primary or secondary by checking liver enzymes and bilirubin levels.

Several 16 to 20 month old Holstein dairy heifers which have been out in pasture have developed large areas of skin sloughing, which appears to affect mainly the white unpigmented areas (see photo). Based on this observation, what is the best diagnosis? A. Insect hypersensitivity B. Ordinary sunburn C. Allergic dermatitis D. Malignant catarrhal fever, skin form E. Photosensitization

B - Escherichia coli E. coli causes the characteristic lesions of edema disease in recently weaned piglets. Marked swelling of the periocular region, forehead and submandibular area follow infection. Piglets may die peracutely. Usually only a few piglets in a group are affected, but affected piglets perish rapidly (within 12 hours). Hemolytic E. coli that produce F18 pili and Shiga toxin 2e are implicated in edema disease. To make a definitive diagnosis, E. coli must be first isolated and then characterized as an edema disease strain (that is, producing F18 pili and Shiga toxin 2e). The course is so rapid that treatment is ineffective. Antibiotics may be administered to unaffected pigs in the group. Brachyspira hyodysenteriae causes Swine dysentery also called bloody scours. Lawsonia intracellularis causes Porcine proliferative enteritis (diarrhea, often with fibronecrotic casts). Clostridium septicum is the agent of Malignant edema in many animals. Infection occurs through contaminated wounds and turns affected muscle dark brown or black. Streptococcus suis causes septicemia and meningitis in weaners and growing pigs

Several piglets in a group weaned 10 days ago in the nursery facility of a large commercial swine operation were found dead. On evaluation, some weaners have swelling around the eyes and forehead. Some are in lateral recumbency and dyspneic. Necropsy of the dead piglets reveals subcutaneous and submucosal edema. The most likely causative organism is... A - Brachyspira hyodysenteriae B - Escherichia coli C - Lawsonia intracellularis D - Clostridium septicum E - Streptococcus suis

A - Hypokalemic myopathy. The posture is classic for hypokalemia and other neuromyopathies (e.g., myasthenia gravis, organophosphate intoxication) in cats. Hyperkalemic periodic paralysis has not been reported in cats.

The 12-year-old spayed female cat shown below is presented with a history of weakness for the past 2 days. She has neck ventroflexion and a stiff, stilted gait. Which one of the following is in the top of the differential list? A - Hypokalemic myopathy B - Hyperkalemic periodic paralysis C - Cervical vertebral malformation D - Caudal occipital malformation E - Bilateral otitis media/interna

This is anterior lens luxation. Note the aphakic crescent to right side (area of pupil without lens). Look for an opaque (cloudy, milky) lens behind the pupil with cataracts. In contrast, nuclear sclerosis looks like this. Orbital cellulitis presents in large and hunting breeds of dogs with acute pain on opening mouth, eyelid swelling, unilateral prolapse of nictitating membrane, eye protrusion, and conjunctivitis. The dog's face will look swollen with inflamed, red eyes (a.k.a.: "Devil dog").

What is wrong with this picture? DV- cat with some weird looking eyes A - Cataract B - Posterior synechiae C - Nuclear sclerosis D - Orbital cellulitis E - Anterior lens luxation

C. Type I. Type I hypersensitivity is IgE-mediated and results in immune mediated hypersensitivity. Type II hypersensitivity is an antibody-dependent reaction and occurs due to IgG or IgM made against normal self antigens or some foreign antigen that resembles some molecule on the surface of host cells. Type III hypersensitivity is an immune complex-mediated reaction. This is caused when soluble antigen-antibody complexes form in large amounts and overwhelm the body instead of being normally removed by macrophages in the spleen and liver. Type IV is considered a delayed hypersensitivity and is cell-mediated. T8-lymphocytes will be sensitized to an antigen and differentiate into cytotoxic T-lymphocytes. T helper 1 type T4-lymphocytes become sensitized to an antigen and produce cytokines.

The dog in the photograph was vaccinated earlier in the day. What type of allergic reaction is occurring? A. Type II B. Type IV C. Type I D. Type III

C. Disruption of function of the plumage Feathers serve a critical waterproofing and insulatory function which is disrupted by oil and can rapidly result in hypothermia. Other concerns for oiled birds include GI irritation from ingestion of oil during preening, hemolytic anemia, and pneumonia due to inhalation of oil. Treatments include heat, supportive care, and activated charcoal. Once stabilized, frequent high pressure, warm, mild detergent baths and clean warm water rinses until water beads freely off of the feathers is important. Birds should be placed in warm air flow until dry and they should be maintained on self-skimming ponds for several days after washing to ensure full waterproofing.

The goose shown in the image below was found weak on the shore of a local pond where oil had been dumped. The goose was covered in oil, dehydrated and weak but responsive. Which of the following is an important acute clinical effect of oil on affected birds? Image used with permission, from Avian Medicine (Forbes), courtesy of Manson Publishing. A. Hepatotoxicity B. Lead toxicity C. Disruption of function of the plumage D. Nephrotoxicity E. Contact dermatitis

B - 5-15% Retained placenta occurs in 5-15% of dairy cattle. Very common, about 1 in 10 calvings. NORMALLY, the placenta is expelled in 3-8 hours. If it's still there 12-24 hours after calving, it's retained. Typically, retained placentas are dispelled spontaneously after 4-10 days as the caruncles necrose. An owner may insist you pull it out. Manual removal is usually not helpful-should only be done if placenta comes out with GENTLE traction. Manual removal is contraindicated if cow has signs of septicemia- removal may cause a septic metritis, peritonitis.

What percentage of dairy cows have a retained placenta after calving? A - 1-4% B - 5-15% C - 25% D - 20-30% E - 30-40%

A - Moraxella bovis In cattle, Moraxella bovis is the most commonly recognized cause of infectious keratoconjunctivitis, ("Pinkeye"); Other causes include Mycoplasma spp. and Neisseria spp. The severity of infection with Moraxella bovis may increase with infection with IBR or other microbes. Chlamydophila pecorum is the most common cause of pinkeye in sheep.

This is a corneal abscess secondary to infectious keratoconjunctivitis (pinkeye). What is the most commonly recognized organism that causes pinkeye in cows? Picture of eye A - Moraxella bovis B - Chlamydophila pecorum C - Mycoplasma spp. D - Colesiota conjunctivae E - Neisseria spp.

MCF virus is ovine herpes virus-2 (OvHV-2) and is carried by 95-99% of sheep in North America which show no symptoms. It is also carried by 75% of domestic goats, 40% of muskox, 37% of bighorn sheep, 25% of pronghorn antelope, 62% of mouflon sheep, and by a small percentage of elk, mule deer, and white tailed deer. Susceptible hosts include cattle, water buffalo, deer, pigs, and bison; bison are the most susceptible.

What species is the principle carrier and vector of Malignant Catarrhal Fever (MCF) virus in North America? A. Sheep B. Horse C. Donkey D. Bison E. Cattle

D - Blackleg This is blackleg, caused by Clostridium chauvoei (feseri). Look for peracute, febrile disease. May have acute lameness and a few animals found dead. Some may have edematous, crepitant ("popping, crackling" sound/feel) swellings. Death within 12-48 hrs. Beef breeds mostly, best animals in herd, 6 mos-2 yrs, summer and fall, rarely in winter. The blackleg organism is a normal inhabitant of gut, infection develops WITHOUT wounds, though bruising may precipitate disease. In contrast tetanus Clostridium tetani, which enters via a wound, and causes disease more typically an individual sick cow, not an outbreak. DDX includes: Anthrax (Bacillus anthracis, reportable) Lightning strike (sudden death, single animal) Bacillary hemoglobinuria (Clostridium hemolyticum) Malignant edema (Clostridium septicum)

Two of the best-growing 14-month old steers at a free range beef operation die suddenly. The farmer reports that one was acutely lame, trembling and prostrate for a half-day before death. The other animal died suddenly. A few other cattle are febrile and lame. On necropsy the following lesions are evident. Click here to see image What is the diagnosis? A - Tetanus B - Bacillary hemoglobinuria C - Malignant edema D - Blackleg E - Infectious necrotic hepatitis

A - Rare disease, Early diagnosis improves prognosis You need a very sensitive test if: 1. Disease is rare (ie: BSE), or 2. Early Dx improves prognosis (ie: HIV in people), or 3. The disease is highly lethal or consequences of missing a case are severe. (ie: Rabies, Brucellosis, BSE, Screw-worm, FMD, EIA) Remember that a HIGHLY SENSITIVE test will have very FEW false negatives. That means if a test is highly sensitive, you can TRUST a NEGATIVE TEST. This sounds contradictory, but it makes more sense if you review this sensitivity diagram. Sensitivity=a/(a+c). "a" are true positives. "c" are false negatives. If sensitivity is HIGH then "c" (FALSE negs) must be small. Therefore, high sensitivity means you can really trust a NEGATIVE result to be correct.

Under what conditions is a very sensitive test used? A - Rare disease, Early diagnosis improves prognosis B - Lethal disease, Highly prevalent disease C - Treatment does not affect prognosis, Non-infectious diseases D - Common disease, infectious diseases E - Highly prevalent disease, Treatment does not affect prognosis

C - Interruption of inhibitory neuron input from lumbar spinal cord With severe trauma between T2-T13, inhibitory pathways (esp. around L2-L4) are interrupted and cervical intumescence neurons (C6-T2) are "released", causing extensor hypertonia in the forelimbs. Think of Schiff-Sherrington syndrome when you see a combination of forelimb extensor rigidity and hind limb flaccid paralysis, in an animal that has just had major spinal trauma, like being hit by a car.

What is the neuroanatomic cause of extensor rigidity in the forelimbs of a cat with Schiff-Sherrington syndrome? A - Subdural white matter compression at C6-T3 B - Brachial plexus trauma C - Interruption of inhibitory neuron input from lumbar spinal cord D - Polyradiculoneurtitis of peripheral nerve sheaths at cervical intumescence E - Damage to thoracic spinocerebellar tracts in superficial white matte

D. Hardbag CAEV is a retrovirus and when it affects the udder it will cause fibrosis and result in a firm udder with agalactia. Treatment is ineffective and the goat should be culled. The disease is usually subclinical but can cause arthritis in adults and encephalitis in kids.

What is the potential udder fate of goats infected with caprine arthritis encephalomyelitis virus? A. The udder is not affected B. Bluebag C. Gangrene D. Hardbag

D. Via mosquitoes Mosquito vectors are the primary mode of transmission for WNV between birds and other hosts (horses, humans). There are infrequent documented cases of the disease being spread by feces or saliva. No mites have been documented to transmit the disease. Birds do not have a placenta.

What is the primary mode of transmission of West Nile Virus (WNV) between birds? A. Via Cnemidocoptes mites B. Via feces C. Via saliva D. Via mosquitoes E. Via placenta

E - Clostridium difficile A history of recent antimicrobial therapy is common in cases of Clostridium difficile associated diarrhea. Adult horses exposed to erythromycin are particularly at risk for C. difficile enterocolitis. Clostridium novyi is the cause of infectious necrotic hepatitis, which is primarily seen in sheep but can also be seen in cattle, hogs, and horses. Lawsonia intracellularis is the cause of proliferative enteropathy, resulting in diarrhea and hypoproteinemia in foals and swine. Rhodococcus equi is a notable cause of pneumonia in older foals characterized by pulmonary abscessation as well as some extrapulmonary manifestations. Escherechia coli can be a cause of septicemia and diarrhea in foals and calves.

When a foal is being treated with erythromycin (for Rhodococcus equi for example), the mare is at risk of developing enterocolitis due to which one of the following organisms? A - Escherechia coli B - Clostridium novyi C - Lawsonia intracellularis D - Rhodococcus equi E - Clostridium difficile

D. Ileocolic artery The correct answer is ileocolic artery. The site for colonic resection is limited by tension on the ileocolic artery when trying to suture your new end of colon to the rectum. Sometimes the tension is too great and instead of a colocolic anastomosis, an ileocolic anastomosis must be performed. Essentially you are trying to connect a section of ascending colon to the rectum. Now that can be pretty far! Performing an ileocolic anastomosis is not ideal because you eliminate the ileocecal valve, and that may predispose the animal to bacterial overgrowth. The caudal mesenteric artery gives branches to the rectum and descending colon. The left colic artery also feeds the descending colon. The pudendal artery supplies the external genitalia. The ileocolic artery provides blood supply to the ascending and transverse colon.

When performing a subtotal colectomy on a feline patient, what blood vessel limits the amount of colon that you are able to remove? A. Pudendal artery B. Caudal mesenteric artery C. Left colic artery D. Ileocolic artery

C - Inflammation of the stifle leading to degenerative joint disease Gonitis is an inflammation of the stifle leading to degenerative joint disease. In cases of glaucoma you might use gonioscopy to classify the glaucoma, detect iridocorneal outflow changes as the disease progresses and help determine the most appropriate medical and surgical treatments. Click here to see the classic big blue eye of acute glaucoma. Click here to see chronic glaucoma with lens luxation, a common sequella. Infection of the crop and sinuses with Trichomonas gallinae is trichomonosis, a protozoal disease of birds. Click here to see a necropsy image of a pigeon with trichomonosis.

Which description correctly matches gonitis? A - Inflammation of the mesorchium, parietal tunic and testicle B - Avulsion of the extensor process of the third phalanx C - Inflammation of the stifle leading to degenerative joint disease D - Infection of the crop and sinuses with Trichomonas gallinae E - Progressive unilateral or bilateral wide-angle glaucoma

A. Atracurium Atracurium is a non-depolarizing neuromuscular blocker. Other neuromuscular blockers include pancuronium, d-tubocurarine and succinylcholine. Succinylcholine is a depolarizing neuromuscular blocker. Fentanyl is an opioid. Naloxone is an opioid antagonist. Edrophonium and neostigmine are cholinesterase inhibitors used to reverse neuromuscular blockers.

Which of these drugs is a neuromuscular blocker when given to a dog? A. Atracurium B. Naloxone C. Neostigmine D. Edrophonium E. Fentanyl

A. Dopamine Adrenergic vasopressors are frequently used to treat hypotension during anesthesia. Drugs in this category are dopamine, dobutamine, ephedrine, phenylephrine, and norepinephrine. Atropine and glycopyrrolate are anticholinergic drugs. Hydralazine is a vasodilator used to treat hypertension.

Which of these is an adrenergic vasopressor in dogs? A. Dopamine B. Hydralazine C. Atropine D. Glycopyrrolate

C. Hypoglycemia The correct answer is hypoglycemia. Hypoglycemia secondary to insulinoma is the most common cause of seizures in the adult ferret. Keep in mind, a prolonged seizure can actually cause hypoglycemia, so the finding of low blood glucose in a seizuring ferret does not necessarily confirm a diagnosis of insulinoma. Idiopathic epilepsy has not been reported in ferrets. Hypocalcemia and hepatic encephalopathy can cause seizures, but are not as commonly reported as hypoglycemia.

Which of these is the most common cause of seizures in the adult ferret? A. Hepatic encephalopathy B. Hypocalcemia C. Hypoglycemia D. Intracranial neoplasia E. Idiopathic epilepsy

A. Tetracycline

Which one of the following antibiotics is the treatment of choice in the face of a vibriosis abortion storm? A - Tetracycline B - Aminoglycoside C - Cephalosporin D - Fluoroquinolone E - Chloramphenicol

B - Tongue hanging out, foal cannont retract it A nonretractable tongue is a cardinal and early sign of toxicoinfectious botulism, (Shaker foal syndrome). Affected foals demonstrate signs of progressive symmetric motor paralysis, including stilted gait, muscular tremors, and recumbency.. Mortality is higher than 90%.

Which one of the following choices is a cardinal and early sign of toxicoinfectious botulism in a foal? A - When startled, affected foals fall and exhibit opisthotonos B - Tongue hanging out, foal cannot retract it C - Sawhorse stance D - Risus sardonicus with protruding third eyelids E - Hemorrhagic enterocolitis with headpressing

E - Proton pump inhibitor Omeprazole is a proton pump inhibitor that decreases gastric acid secretion. Omeprazole inhibits the sodium/potassium proton pump at the luminal surface of parietal cells. Parietal cells normally secrete hydrogen ions into the stomach, a key component of acidic HCL. Other drugs that decrease gastric acid secretion include the H2-receptor antagonists cimetidine, ranitidine, and famotidine and a synthetic prostaglandin E1 analog called Misoprostol. Carprofen, Etodolac, Deracoxib, Meloxicam and Firocoxib are all nonsteroidal anti-inflammatory drugs (NSAIDS) that may CAUSE gastric acid secretion.

Which one of the following choices is the mechanism of action of omeprazole? A - Cyclooxygenase blocker B - Beta-adrenergic receptor agonist C - Synthetic prostaglandin E1 analog D - H2-receptor antagonist E - Proton pump inhibitor

C - White muscle disease, Enterotoxemia, Colibacillosis. When you hear "Sudden Death" in calves, think of white muscle disease caused by Selenium/vitamin E deficiency. Also think of enterotoxemia caused by Clostridium perfringens type B or C. And think of severe colibacillosis. Can also see sudden death with encephalitic form of IBR and severe salmonellosis.

Which three diseases are on the differential diagnosis list when encountering sudden death in a young, fast-growing calf? A - Lasalocid toxicity, Salmonellosis, Lymphosarcoma-juvenile form B - Salmonellosis, Colibacillosis, Enzootic Calf Pneumonia C - White muscle disease, Enterotoxemia, Colibacillosis D - Bovine Viral Diarrhea, Lasalocid toxicity, White muscle disease E - Winter dysentery, Enterotoxemia, IBR-encephalitic form

D. Vesicular stomatitis VS affects horses, cattle and pigs with similar clinical signs. It is a viral disease of high morbidity and low mortality that appears about every 7 to 10 years in the Southwestern United States. Yearly outbreaks occur in southern Mexico and northern South America. Black flies and midges appear to be the vectors. Affected premises should be quarantined.

You are called one hot summer day to see a group of horses in New Mexico which are slobbering and not eating their hay. Three out of 20 horses seem to be visibly affected. On physical exam of the first one, you find fever of 104 F (40 C) and obvious oral ulcers as shown in the image, mainly on the tongue. What is your tentative diagnosis? A. Bovine papular stomatitis B. Foot-and-Mouth disease (FMD) C. African Horse Sickness D. Vesicular stomatitis E. Glanders

A - Canine herpes virus Canine herpes virus (CHV) rarely causes disease other than mild respiratory signs in adult dogs. However, naïve newborn pups have high morbidity and mortality if infected at less than 3 weeks of age. Naïve bitches should be exposed to CHV prior to breeding or the bitch and pups quarantined for 3 weeks before and after whelping. Canine parvovirus in neonates can cause vomiting, diarrhea, or acute death due to cardiovascular failure in pups 3-8 weeks old.

You are presented with a bitch that whelped her first litter 2 weeks ago. Two of the pups have died and the remaining 2 have neurological signs, suckle poorly, and cry constantly. Which one of the following choices is the primary differential diagnosis? A - Canine herpes virus B - Canine parvovirus C - Neonatal septicemia D - Transplacental roundworm infection E - Congenital disease

E. Coliform mastitis This is a case of severe acute coliform mastitis, and the absorbed endotoxin (LPS) is causing many of the systemic signs observed. The cow needs to be aggressively treated with IV fluids, NSAIDS, and supportive nursing. The gland should be frequently milked out. The use of both intramammary and systemic antimicrobial drugs to which most coliforms are susceptible is still controversial, but is often done in cows in a severe state of illness as in this case. While this cow may have secondary hypocalcemia, treatment with calcium needs to be approached cautiously, as endotoxic animals have very sensitive myocardium and arrest may occur if calcium is given IV. If given, preferred routes of calcium administration would be subcutaneous or oral.

You examine a very ill 4-year old Holstein dairy cow on a large commercial dairy. She freshened one week ago and was producing well, until she was found down and unwilling to rise this morning when you were called. T=103F or 39.4 C, HR=90, and RR=35. The scleral vessels are dark are enlarged, her rumen is fairly empty and the motility is poor, and she appears too weak to rise. Rectal exam reveals an involuting uterus which can be retracted, discharging a brownish red mucoid non-odorous lochia through the vagina. The left rear quarter of her udder is swollen, hot, painful, and discolored (see image), and contains a serum-like secretion with clumps of fibrin in it. What is your diagnosis? A. Grain overload B. Metritis C. Displaced abomasum D. Hypocalcemia (milk fever) E. Coliform mastitis

D. Empty the litter box daily as a simple precaution to prevent infection as it takes 1-3 days for passed oocyts in the stool to sporulate into an infective form Toxoplasmosis gondii is a protozoal organism. The cat is the definitive host; the entire life cycle of the organism can be completed within this host. Most cats become infected when they consume an exposed rodent with bradyzoites encysted in their tissues. Only recently infected cats generally shed oocysts in their stool, and cats typically only shed these oocysts for 1-2 weeks. Most cats will only have one shedding episode in their lifetime. A IgM (not IgG) titer of 1:64 or greater suggests recent or active infection and that cat is at risk of shedding oocysts in their stools. Oocyts are not infective until they sporulate. This process takes > 24 hours, so emptying the litter box daily is advised, preferably by someone who is not pregnant. If an owner has owned cats for a long while, it is possible that they may have previously been exposed and therefore have mounted an immune response to the organism. If so, it may be advisable to test for Toxoplasma antibody titers in the owner. A sufficient antibody titer will mean the client is protected from infection during the first trimester.

Your client is pregnant and is worried about acquiring toxoplasmosis from her cat. What do you advise? A. Submit toxoplasmosis titers from the cat. A IgG titer of 1:64 or greater suggests recent or active infection that could pose a danger. B. Submit toxoplasmosis titers from the cat. A positive cat infected with toxoplasmosis can shed multiple times in its lifetime and pose a zoonotic risk. C. Toxoplasmosis titer should be performed on the owner by a human physician. A positive titer indicates antibodies to the organism that will prevent infection in the first trimester. A negative titer indicates she should remove her cats from her environment. D. Empty the litter box daily as a simple precaution to prevent infection as it takes 1-3 days for passed oocyts in the stool to sporulate into an infective form


Related study sets

Marketing Chapter 4 - Consumer Behavior

View Set

5: Development and plasticity of the brain

View Set

Survey of Operating Systems (Linux)

View Set

PN Case Study Loss Grief and Death

View Set

Suspense in "The Cruel Tribute" Quiz

View Set

Uipath : Data Manipulation with Strings in Studio

View Set

Chapter 31: Drug Therapy for Nasal Congestion and Cough

View Set

Math Definitions, Axioms, and Properties

View Set

Congress Powers and the Commerce Clause and the Dormant Commerce Clause

View Set